SlideShare una empresa de Scribd logo
1 de 170
Descargar para leer sin conexión
Pediatric Endocrinology Review
MCQs
Abdulmoein Eid Al-Agha, FRCPCH
Professor of Pediatric Endocrinology,
Website: http://aagha.kau.edu.sa
Five-year-old girl presented with bilateral breast
enlargement & vaginal discharge, together with
moodiness and body odor, no relevant past
medical history. She was well with no headaches,
visual disturbance or polydipsia. Mother and two
elder sisters had early menarche at 10–11 years.
On examination, her height is on the 90th% and
mid-parental height 50th%. Tanner stage is of B3,
PH2, A1. Which one of the following is most
important investigation?
a) Observation of further progression of pubertal signs.
b) Bone age assessment.
c) MRI pituitary to look for CNS tumor.
d) Basal &GnRH stimulation test.
Precocious Puberty
• Central precocious puberty (CPP) is caused by an
early activation of the hypothalamic-pituitary-
gonadal axis.
– CPP is pathologic in up to 40 - 75 % of boys & 10 - 20
% of girls.
• Peripheral precocity is caused by:
– secretion of sex hormones either from the gonads or
adrenal glands, ectopic human chorionic
gonadotropin (hCG) production by a germ-cell tumor,
or by exogenous sources of sex steroids
– Is independent from the hypothalamic-pituitary-
gonadal axis
Copyrights apply
Five- year old girl, brought by her mother because of
bilateral breast enlargement and spotty vaginal
discharges. On examination (see photo). Her basal
pubertal investigations revealed:
- Estradiol 62 pg/ml (<10)
- FSH <0.1 mIU/mL
- LH <0.1 mIU/mL
Which one of the following, is the most
important confirmatory investigation you
will order?
a) GnRH stimulation test.
b) Gene mutation screening.
c) Thyroid function test.
d) Skeletal survey.
McCune-Albright syndrome (MAS) consists of at least 2
of the following 3 features:
• Polyostotic fibrous dysplasia (PFD).
• Café-au-lait skin pigmentation,
• Autonomous endocrine hyperfunction (e.g.,
gonadotropin-independent precocious puberty).
• Other endocrinopathies may be present, including
hyperthyroidism, acromegaly &Cushing syndrome.
• Genetically, activating mutation of the GNAS1 gene ,
which is involved in G- protein signaling .
Copyrights apply
Copyrights apply
A 2 year old girl with bilateral breast development with
no growth acceleration, no bone age advancement and
normal estradiol, LH or FSH. What is the most likely
diagnosis ?
a) Ingestion of her mother’s OCPs.
b) Precocious puberty.
c) Benign isolated premature thelarche.
d) McCune Albright Syndrome.
Benign Premature Thelarche
• Isolated breast development
– 80% before age 2 years.
– Rarely after age 4 years.
• Not associated with other signs of puberty.
(growth acceleration, advancement of bone age)
• Children go on to normal timing of puberty and
normal fertility.
• It may be associated with functional follicular cysts
that spontaneously regress and perhaps with
especially responsive breast tissue
• Benign process.
• Routine follow-up.
Five- year old girl, with 6 months history of pubic hair growth
associated with fine axillary hair as well as adult odor to
sweat. No breast development with no acceleration of
growth. Otherwise normal history and examinations.
What is the most likely diagnosis?
a) Precocious puberty.
b) Benign premature Adrenarche.
c) Non-classical congenital adrenal hyperplasia.
d) Adrenal tumor.
Benign premature adrenarche
• Production of adrenal androgens before true
pubertal development begins.
• Presents as isolated pubic hair in mid childhood
– No growth acceleration.
– No testicular enlargement in boys.
• If normal growth rate, routine follow-up.
• If accelerated growth and/or bone age advancement,
screen for:
– CAH
– Virilizing tumor (adrenal/gonadal)
Four- year old girl, presented with abdominal
distension, there is A three months history of bilateral
breast enlargement, pubic hair appearances & rapid
growth. Her abdominal ultrasound (photo).
Which one of the following is most likely diagnosis?
a) β-HCG secreting hepatoblastoma
b) McCune Albright syndrome.
c) Benign premature Adrenarche.
d) Ovarian cyst.
Gonadotropin independent precocious
puberty secondary to β-HCG secreting
hepatoblastoma
Hepatoblastoma
• Is the most common primary hepatic malignancy in early
childhood.
• The majority of cases occur in the first two years of life and rarely
in children older than five years.
• Syndromes with an increased incidence of hepatoblastoma
include:
– Beckwith Wiedemann syndrome.
– trisomy 18 & trisomy 21 .
– Aicardi syndrome.
– Li-Fraumeni syndrome.
– Goldenhar syndrome.
– type 1a glycogen storage disease.
– familial adenomatous polyposis.
• Serum alpha-fetoprotein (AFP) levels are markedly elevated.
• Sexual precocity may be present due to the synthesis of ectopic
gonadotropin (HCG).
Thirteen-year-old girl, is referred with growth failure
with delayed puberty. On examination her height is
below the 0.4th centile and her weight is on the 25th
centile. (Please see photo).
Which one of the following is most likely diagnosis?
a) Constitutional delay of Puberty.
b) Hypogonadotropic hypogonadism.
c) Hypopituitarism.
d) Hypergonadotropic hypogonadism.
Ovarian failure
• Turner syndrome is one of the most common causes of
premature ovarian failure.
• Most affected girls have no breast development and
have primary amenorrhea.
• Approximately 15 - 30 % of girls with Turner syndrome
either have initial breast development followed by
pubertal arrest, or complete puberty but then develop
secondary amenorrhea.
• A small percentage of girls have normal pubertal
development and regular menstruation.
• These milder phenotypes of ovarian failure are more
common in girls with mosaicism compared with those
with 45,X monosomy.
A full term neonate, is born with isolated bilateral swollen
feet. What is the most likely diagnosis?
a) Cardiac failure with lower limb edema.
b) Systemic allergic reaction.
c) Congenital nephrotic syndrome.
d) Turner syndrome.
Turner syndrome
• Combination of short stature, primary amenorrhea
(ovarian dysgenesis), webbed neck, lymphedema, and
cubitus valgus.
• incidence among live born female infants of one in
5000.
• More than half have a 45, X karyotype.
• The remainder show mosaicism and/or more complex
rearrangements involving the X chromosome.
• Between 20% and 40% of girls with Turner syndrome
have significant heart defects, most commonly
coarctation of the aorta (70%), often bicuspid aortic
valve, and aortic stenosis.
Copyrights apply
Ten –year - old girl, presented with arm
deformity (photo). What is the diagnosis?
Madelung deformity
• Madelung deformity is a focal dysplasia of the
distal radial epiphysis.
• Premature closure leads to a progressive
deformity with dorsal displacement of the
ulna.
• Madelung deformity occurs in girls with Turner
syndrome and is usually bilateral.
• The diagnosis of Madelung deformity is made
radiographically; radiographs of both wrists,
forearms, and elbows should be obtained.
Copyrights apply
Seven-year -old boy has presented to the
endocrinology clinic with short stature & mental
subnormality. What is the most likely diagnosis?
a) Turner’s syndrome.
b) VATER syndrome.
c) Silver – Russel syndrome.
d) Noonan’s syndrome.
Noonan syndrome (NS)
• Autosomal dominant disorder.
• The classical features include:
– short stature.
– congenital heart disease (CHD).
– clinical features of NS include difficulties with
feeding in early life; vision, hearing, and growth
problems; specific learning difficulties & easy
bruising and bleeding.
Ten- year female presented, with a 4 week history of
polyuria, polydipsia, and unexplained weight loss. She
was noticed to have deep, sighing respiration. Glucose
was 498 mg/dL, pH 7.06. Which one of the following is
the most important initial management ?
a) Insulin drip 0.1 units/kg/hour.
b) ½ Normal Saline with 40 meq K at 2x maintenance.
c) Bicarbonate 1 meq/kg slowly over 1 hour.
d) Fluid rehydration with 0.9 NS.
Sixteen -year old boy, presented with delayed puberty.
He was having a recurrent episodes of headache,
diplopia and increased urination. His height was < 3rd
percentile. Which one of the following is the most likely
diagnosis?
a) Diabetes mellitus.
b) Cerebellar tumor.
c) Craniopharyngioma.
d) Kalman's syndrome.
Craniopharyngioma
• Benign tumor with tendency to recur after excision and the
high surgical risk due to involvement of the most vital
structures of the brain.
• Rare solid or mixed solid & cystic tumors that arise from
remnants of Rathke's pouch.
• Historically, been named as "Rathke pouch tumors“.
• A wide range of symptoms may be present.
• Visual symptoms are frequent, result from compression of
the optic chiasm or nerves.
• Moderate to severe daily headaches are present in
approximately 50 % of patients at the time of diagnosis.
• Other generalized symptoms, such as depression, nausea,
vomiting, and lethargy can accompany pressure-related
headaches.
Craniopharyngioma
• Endocrine abnormalities due to direct damage to or
compression of normal structures or post surgical
removal of the tumor can lead to a range of
endocrine abnormalities.
• Frequently observed complications include diabetes
insipidus (75%).
• Growth failure, caused by either hypothyroidism or
growth hormone deficiency, is the most common
presentation in children.
• Delayed puberty could be seen in 40 % of cases.
Copyrights apply
Twenty – day- old, neonate was seen in the pediatric
endocrinology clinic for recurrent episodes of
hypoglycemia . On examination, he had a cleft lip and
palate with a small mid-face. Both testes were
palpable, however short penile length was
measured. What is your best approach in order to
reach to the final cause of hypoglycemia?
a) Look for other dysmorphic features.
b) Admit to do critical samples during his
hypoglycemia attack.
c) Do GH stimulation test.
d) Do MRI brain.
Holoprosencephaly (HPE)
• Is cephalic disorder in which the forebrain of the
embryo fails to develop into two hemispheres.
• Normally, the forebrain is formed and the face begins
to develop in the fifth and sixth weeks of gestation.
• The condition can be mild or severe.
• Most cases are not compatible with life and result in
fetal death in-utero.
• When the embryo's forebrain does not divide to form
bilateral cerebral hemispheres (the left and right halves
of the brain), it causes defects in the development of
the face , brain & pituitary structure and function.
Hypopituitarism
• Affects between 1 in 4,000 -10,000 live births,
with increasing incidence with age.
• Congenital hypopituitarism most often results
from genetic or embryologic pathologies.
• The clinical presentation of hypopituitarism,
widely varies, depends on the patient's age,
the etiology, and the specific hormone
deficiencies, which may occur as isolated
deficiencies or in various combinations of
MPHD.
Hypopituitarism
• Most neonates with hypopituitarism have
normal birth weights and lengths and no history
of intrauterine growth retardation.
• However, they often have histories of breech
presentation (particularly neonates with MPHD),
although the explanation for this is unclear.
• The hypoglycemia risk is higher in neonates with
hypopituitarism, with various manifesting
symptoms, such as lethargy, jitteriness, pallor,
cyanosis, apnea, or convulsions.
• Jaundice may be secondary to indirect
hyperbilirubinemia (TSH deficiency) or to direct
hyperbilirubinemia (GH or ACTH deficiencies).
Ten-day old boy presented with hypertension &
metabolic alkalosis. He was diagnosed with
congenital adrenal hyperplasia. Which of the
following enzyme deficiency could be the cause?
a) Aldosterone synthase deficiency.
b) 21-β- hydroxylase deficiency.
c) 3-β-hydroxysteroid dehydrogenase
deficiency.
d) 17- hydroxylase deficiency.
• 17-Hydroxylase (17-OH) deficiency syndrome is a
rare genetic disorder of steroid biosynthesis .
• It causes decreased production of glucocorticoids
and sex steroids, resulting in 46,XY DSD.
• Increased synthesis of mineralocorticoids
precursors, resulting in varying degrees of
hypertension and hypokalaemia
• It is a rare form of congenital adrenal hyperplasia
resulting from loss-of-function mutations involving
the CYP17 gene.
• Exogenous glucocorticoid therapy is the treatment
of choice which suppresses adrenocorticotropic
hormone (ACTH) secretion, decreases 11-DOC and
corticosterone levels, and normalizes serum
potassium and blood pressure.
A neonate was born with abnormal newborn thyroid
screen done at 3 days of age: fT4 was 8 pmol/l (12-22).
TSH was 60 μIU/mL (0.3-5.0). Which one of the
following might be the cause?
a) Hypothyroidism due to dysgenesis of the thyroid
gland.
b) Central hypothyroidism.
c) TBG deficiency.
d) Hypothyroidism from excess iodine exposure.
• Thyroid dysgenesis/agenesis is the commonest cause of
congenital primary hypothyroidism in almost 90% of cases.
• Prevalence 1 in 4,000.
• Symptoms and signs include the following:
– Decreased activity.
– Large anterior fontanelle.
– Poor feeding and weight gain.
– Small stature or poor growth.
– Jaundice.
– Decreased stooling or constipation.
– Hypotonia.
– Hoarse cry.
6 month female with
congenital hypothyroidism
..following 4 months
therapy
Thirteen -year-old - girl presented to the orthopedic
clinic with two year history of a limping in her left leg.
She has been diagnosed with hypothyroidism but not
compliant on her medication. On examination she is
short & obese with a goiter and other signs of
hypothyroidism. She had limitation of movement of her
left hip and a limp. What is the most likely diagnosis?
a) Slipped capital femoral epiphysis (SCFE).
b) Chronic Osteomyelitis.
c) Vitamin D deficiency.
d) Monoarticular rheumatoid arthritis.
Slipped capital femoral epiphysis (SCFE)
• Characterized by a displacement of the capital femoral
epiphysis from the femoral neck through the physeal plate.
• The mean age of presentation is 12 years in girls and 13.5
years in boys, near the time of peak linear growth.
• Obesity is a significant risk factor.
• The two most common features of the presentation are
pain and altered gait.
• The classic presentation is that of an obese adolescent with
a complaint of chronic or intermittent non radiating, dull,
aching pain in the hip, groin, thigh, or knee, and no history
of preceding trauma.
• The diagnosis is usually made on plain radiographs, which
reveal an apparent posterior displacement of the femoral
epiphysis, like ice cream slipping off a cone.
Slipped capital femoral epiphysis (SCFE)
Newborn with DSD, noted to have generalized
hyperpigmentation. Pelvic ultrasound demonstrated the
presence of a both ovaries &uterus. What is the most
useful test for the diagnosis?
a) Testosterone.
b) 17-hydroxyprogesterone.
c) Serum electrolytes.
d) DHEAS.
Congenital adrenal hyperplasia
• It is autosomal recessive disorders.
• Over 95 % of cases are due to 21-hydroxylase deficiency.
• In many countries, neonatal screening for 21OHD is
performed routinely in all newborns by measurement of
17OHP in a dried, filter paper blood spot.
• The characteristic biochemical abnormality in patients with
classic 21OHD is an elevated serum concentration of 17-
hydroxyprogesterone (17OHP).
• False-positive results are common with premature infants,
and many screening programs have established reference
ranges that are based upon weight and gestational age.
• False-negative results may occur as a result of maternal
antenatal glucocorticoid use, in first 2 days of life & in sick
neonates.
Cholesterol
Pregnenolone
Progesterone
DOCA
Corticosterone
ALDOSTERONE
17 (OH) pregnenolone DHEA
17 (OH) progesterone Androstenedione
Compound S
CORTISOL
TESTOSTERONE
Desmolase
3--HSD 3--HSD 3--HSD
17-OH
17-OH
21-OH 21-OH
11-OH 11-OH
Types of CAH
• Classical type:
simple virilizing.
salt losing.
• “Non classical” / Late onset.
a) Assure & re-examination him again at age 18 months.
b) Refer the patient for exploratory laparotomy.
c) Measure basal & HCG stimulated testosterone & DHT.
d) Start therapy with testosterone enanthate, 50 mg IM
monthly for 3 months.
Six-month old 46 XY infant presented with non
palpable testes. Which one of the following, the
most appropriate next step would be?
Cryptorchidism
• Usually is an isolated finding.
• However, it may occur in association with endocrine
disorders, genetic syndromes, &morphologic
abnormalities, particularly if the cryptorchidism is
bilateral.
• Disorders of sex development (DSD) (e.g., mixed
gonadal dysgenesis, Ovotesticular disorders of sex
development, persistent Müllerian duct syndrome)
• Bilaterally nonpalpable testes in a phenotypically male
newborn require immediate evaluation at the time of
birth because the differential diagnosis includes DSD.
Fifteen year old, female has presented with
primary amenorrhea. Her breast development
began at 10 years of age. Current height was 160
cm, weight was 45 kg. Tanner stage of B3 PH 1-2.
What is the most likely diagnosis?
a) Mullerian dysgenesis.
b) Turner’s syndrome.
c) Complete androgen insensitivity syndrome.
d) Kalman's syndrome.
Delayed puberty is defined by:
• In boys:
– No enlargement of the testes (testicles) by age 14
years.
– A time lapse of more than 5 years from the onset to
the completion of growth of the genitals.
• In girls:
– No breast development by age 13 years.
– A time lapse of more than 5 years from the beginning
of breast growth to the first menstrual period.
– No menstruation (amenorrhea) by age 16.
• Delayed puberty could be either to hypogonadotropic
hypogonadism or hypergonadotropic hypogonadism.
• The most common cause of delayed puberty is hypogonadotropic
hypogonadism “ functional defect” “ constitutional delay”.
• Other functional defects, such as undernutrition, excessive
exercise, anorexia or chronic illness.
• Adolescents, particularly girls, who become very thin because of
excessive exercise or dieting often have delayed puberty,
including an absence of menstruation.
• Kallmann syndrome is one of genetic disorders, with isolated
gonadotropin deficiency (without affecting production of other
hormones).
• Hypergonadotropic hypogonadism in boys, could be due to
twisting of a testis (testicular torsion), or infection (such
as mumps) or, chromosomal abnormalities, such as Turner
syndrome in girls and Klinefelter syndrome in boys.
Six-year-old girl is referred with growth failure,
poor appetite, recurrent abdominal pain, ‘thick
custard’ stools and vomiting. What is the
appropriate most confirmatory investigation?
a) Bone age.
b) Anti-tissue transglutaminase antibody.
c) Jejunal biopsy.
d) Serum iron & ferritin.
Jejunal biopsy is confirmatory investigation of
celiac disease, which will reveal villous atrophy
& lymphocytic infiltrate in the lamina propria
with hyperplastic crypts.
If a child presented with abdominal distension
& short stature, think of celiac disease !
Celiac disease
• Celiac disease is a small bowel disorder characterized
by mucosal inflammation, villous atrophy, and crypt
hyperplasia, which occur upon exposure to dietary
gluten and demonstrates improvement after
withdrawal of gluten from the diet.
• Children with celiac disease may exhibit no unusual
findings on physical examination.
• Sometimes, protuberant abdomen & marked muscle
wasting and evidence of malnutrition.
• Usually, they responded very well to a strict gluten‐free
diet that she must follow for the rest of her life.
Celiac disease
• Evaluation begins with serologic testing.
• Patients with positive serologic testing, should
undergo an upper endoscopy with small bowel
biopsy to diagnose celiac disease.
• IgA - anti-tissue transglutaminase (tTG) antibody
is the single preferred test for detection of celiac
disease.
• In addition, total IgA levels should be measured.
• In patients with IgA deficiency, IgG-deamidated
gliadin peptide (DGP) IgG.
Eight –year - old girl, has presented with goiter,
abnormal thyroid function test of high TSH, low fT4. She
has sensorineural hearing loss. What is the most likely
inheritance of her disease?
a) Autosomal recessive.
b) Autosomal dominant.
c) Sex-linked dominant.
d) Sex –linked recessive.
Pendred syndrome
• It is inherited in an autosomal recessive.
• Is caused by a mutation in the SLC26A4 gene, which
causes a defect in the organification of iodine (i.e.,
incorporation into thyroid hormone), congenital
sensorineural hearing loss, and, usually, an enlarged
thyroid gland.
• combination of congenital bilateral sensorineural
hearing loss & goiter with euthyroid or mild
hypothyroidism.
• There is no specific treatment, other than supportive
measures for the hearing loss and thyroxine in case
of hypothyroidism.
Twelve-year old girl, who has presented with
exophthalmos and weight loss. Her laboratory
tests confirm Graves disease. Which most likely
cause for her exophthalmos?
a) Fat tissue deposition.
b) Ocular muscle hyperplasia.
c) Inflammatory infiltrate.
d) Globe enlargement.
Graves Ophthalmopathy
• A gritty or foreign object sensation in the eyes.
• Excessive tearing that is often made worse by
exposure to cold air, wind, or bright lights.
• Eye or retroocular discomfort or pain.
• Blurring of vision.
• Diplopia.
• Occasionally decreased vision.
Graves' orbitopathy (ophthalmopathy)
• Is an autoimmune disease of the retroorbital tissues.
• There is no much evidence that RAI causes or worsens
Graves ophthalmopathy in children or adolescents, as
compared to adults.
• children with active, mild orbitopathy may still be
candidates for anti-thyroid, radioiodine, or surgery.
• children with active, moderate-to-severe orbitopathy,
anti-thyroid or surgery are the preferred treatment
options.
• Patients who refuse surgery and who have
contraindications to anti-thyroid may need to be offered
radioiodine therapy with steroid coverage.
Graves' orbitopathy (ophthalmopathy)
• Prednisone seems a better choice than intravenous
corticosteroids, for treating moderate-to-severe Graves’
ophthalmopathy.
• Prednisone (1 mg/kg/day) for four weeks, followed by
gradual tapering over another 2-3 months depending on
severity and response.
• For patients treated surgically, total thyroidectomy is the
procedure of choice.
• Total thyroidectomy eliminates more thyroid antigens than
subtotal thyroidectomy, resulting in progressive decreases in
antibodies to all major thyroid antigens and especially to the
thyrotropin receptor.
• Total thyroidectomy was associated with less progression of
proptosis postoperatively than subtotal thyroidectomy.
Disrupted muscle fibers and an area of inflammatory cell accumulation in the
superior rectus muscle of a patient with ophthalmic Graves' disease. The extra ocular
muscles are swollen, and some muscle fibers show loss of striation, fragmentation,
and infiltration with lymphocytes, most of which are T lymphocytes.
Ten – year old girl with graves disease, on carbimazole
therapy. Which one of the following is the most serious
side effect of anti -thyroid medication?
a) Fever.
b) Rash.
c) Granulocytopenia.
d) Arthritis.
Anti-Thyroid medications
• Used for long-term control of hyperthyroidism in
children, adolescents, and pregnant women
• Methimazole is more potent and longer-acting than
propylthiouracil.
• Propylthiouracil is reserved for use in thyroid storm,
first trimester of pregnancy, and methimazole allergy
or intolerance.
• Patients with Graves disease may experience remission
after treatment for 12-18 months, but recurrences are
common within the following year.
• Toxic multinodular goiter & toxic adenoma will not go
into remission.
Anti-thyroid therapy
• Thionamide compounds were found in 1943 to inhibit
thyroid hormone synthesis.
• Methimazole (MMI) is first choice of anti- thyroid
therapy.
• PTU is associated with risk of severe hepatotoxicity.
• Anti –thyroid medications, inhibit both organification
of iodine to tyrosine residues in thyroglobulin &
coupling of iodothyronines.
• Adverse effects of anti- thyroid therapy include, minor
side effects (e.g., rash, fever, arthritis) and rare serious
adverse effects, such as agranulocytosis &
hepatotoxicity.
• If remission does not occur following ATD
therapy, 131I or surgery should be contemplated.
• When 131I is administered, dosages should be
greater than 150 uCi/gm of thyroid tissue, with
higher dosages needed for large glands.
• When surgery is performed near total or total-
thyroidectomy is the recommended procedure.
• Complications for thyroidectomy in children are
considerably higher than in adults, thus an
experienced thyroid surgeon is needed when
children are operated on.
Eleven- year- old boy presented with severe
proptosis , tachycardia & fine tremor. He is
confirmed to have Graves disease.
Which of the following is most accurate
statement?
a) Radioactive iodine therapy in this case is first
line of management.
b) PTU is more preferable than MTZ.
c) Granulocytopenia is a complication of anti
thyroid medications.
d) Surgery is a good option for this child.
Sixteen- year - old boy presented with thyrotoxicosis
with bilateral, firm, non- pitting, asymmetrical plaques
over his legs (see photo).
Which one of the following describes skin lesion?
a) Pretibial myxedema.
b) Scleroderma.
c) Eczematous lesions.
d) Dermatomyositis.
Pretibial myxedema (PTM)
• PTM or thyroid dermopathy is a term used to describe
localized lesions of the skin resulting from the deposition
of hyaluronic acid, usually as a component of thyroid
disease.
• Thyroid dermopathy occurs rarely.
• Although PTM is most often confined to the pretibial
area, it may occur anywhere on the skin, especially the
ankle, dorsum of the foot, knees, shoulders, elbows,
upper back, pinnae, nose, and neck.
• It is nearly always associated with autoimmune thyroid
disease.
Six- year - old boy with obesity, mental
retardation and subcapsular cataract.
Which one of the following is expected
laboratory finding?
a) Low calcium, low phosphate
b) High calcium, high phosphate
c) Low calcium, high phosphate
d) Normal calcium, normal phosphate
Pseudohypoparathyroidism (PHP)
• In 1942, Fuller Albright first introduced the term
Pseudohypoparathyrodism to describe patients who
presented with PTH-resistant hypocalcaemia and
hyperphosphatemia along with skeletal defects,
collectively termed “Albright hereditary osteodystrophy
(AHO)”.
• These features included short stature, rounded face,
shortened fourth metacarpals of hands and feet, obesity,
dental hypoplasia, and soft-tissue calcifications.
• Results from end organ resistance(renal & bone) to
parathyroid hormone (PTH).
• Characterized by hypocalcemia, hyperphosphatemia, &
elevated PTH level.
Pseudohypoparathyroidism (PHP)
• There are several subtypes of PHP, based upon
genetic and clinical characteristics including.
– type 1a
– type 1b
– type 1c (PHP-1c
– type 2 (PHP-2)
– pseudo pseudohypoparathyroidism (PPHP).
• PHP type 1a is the most common subtype and
represents 70% of cases.
• AHO phenotype is not a feature of PHP-1b or PHP-2.
Thirteen- year old girl with obesity, retinitis
pigmentosa and hypogonadism. Which of the
following is most likely diagnosis?
a) Prader –Willi syndrome.
b) Laurence- Moon- Biedl syndrome.
c) Lowe oculo-cerebral syndrome.
d) Familial cerebellar ataxia.
Genetic syndromes associated with childhood
obesity include the following:
• Prader-Willi syndrome.
• Pseudohypoparathyroidism.
• Laurence-Moon-Biedl (Bardet-Biedl) syndrome
• Cohen syndrome.
• Down syndrome.
• Turner syndrome.
Bardet–Biedl syndrome (BBS)
• A rare autosomal recessive syndrome
• Characterized by retinitis pigmentosa, obesity,
post-axial polydactyly, renal dysfunction,
learning difficulties and hypogonadism.
• The diagnosis is based on clinical findings and
can be confirmed by sequencing of known
disease-causing genes in 80% of patients.
Retinitis pigmentosa
Post axial polydactyly
Eleven- year - old boy, presented with truncal
obesity, striae, hypertension & osteoporosis.
Which one of the following is the commonest
cause?
a) Adrenal adenoma.
b) Adrenal carcinoma.
c) Bilateral adrenal hyperplasia
d) Lipoid adrenal hyperplasia
Copyrights apply
ACTH-independent Cushing syndrome
• Overproduction of glucocorticoid may be due to an
adrenal adenoma, adrenal carcinoma, or macronodular
or micronodular adrenal hyperplasia.
• The zona fasciculate and zona reticularis layers of the
adrenal cortex normally produce glucocorticoid and
androgens.
• Glucocorticoid-secreting tumors are derived from these
cells and, thus, may secrete both glucocorticoid and
androgens.
• In general, excess androgen secretion is suggestive of
an adrenal carcinoma rather than an adrenal adenoma.
• These glucocorticoid-producing tumors do not usually
secrete Aldosterone, which is produced in the zona
glomerulosa layer of the adrenal cortex.
ACTH-independent Cushing syndrome
• Cushing syndrome may be overt, subclinical, cyclical, or
periodic.
• Primary bilateral macronodular adrenal hyperplasia is
uncommon and characterized by multiple nonpigmented
nodules that are greater than 10 mm in diameter and enlarged
adrenal glands.
• The exact etiology of this condition is not quite clear, however,
genetic mutations, paracrine ACTH secretion, and aberrant
hormone receptors have been reported to play a role in its
pathogenesis.
• McCune-Albright syndrome is a rare cause of precocious
puberty.
– It is associated with hyperfunction of the adrenal glands that
may lead to Cushing syndrome.
• Ectopic cortisol secretion from a case of ovarian carcinoma has
been reported as a cause of ACTH independent Cushing
syndrome.
Ten-year old boy with Cushing syndrome
presented acutely in the emergency room with
Addisonian crisis. Which statement is most
accurate?
a) This is impossible, as diagnosis must be
inaccurate.
b) The patient should be reinvestigated again.
c) The patient should be treated as Addisonian
crisis.
d) The patient is likely to have mixed pituitary tumor
causing both diseases.
• The most common cause of Cushing syndrome
in children is prolonged exposure to steroids.
• This often lead to suppression of H-P-A axis, so
patient will not be able to produce cortisol in
stress situations.
• This result in Addisonian crisis if steroid doses
were not increased during stress.
Eleven - year old girl, has presented with
headache, associated with hypertension.
Her electrolytes revealed hypokalemia and
alkalosis. Which one of the following is most
likely diagnosis?
a) Diuretic therapy.
b) Bartter syndrome.
c) Gitelman syndromes.
d) Conn’s syndrome.
Conn’s syndrome
• The classic presenting signs of Conn's syndrome are
hypertension & hypokalemia.
• The most common subtypes of primary Aldosteronism are:
– Aldosterone-producing adenomas (APAs)
– Bilateral idiopathic Hyperaldosteronism (IHA; bilateral
adrenal hyperplasia)
– Less common forms include familial Hyperaldosteronism
(FH) types I to IV.
• Unilateral adrenal hyperplasia.
• Aldosterone-producing adrenocortical carcinoma.
• Ectopic aldosterone-producing tumor.
Sixteen -year old boy presented with
Gynecomastia. Which of the following
medications could be the cause?
a) Hydralazine.
b) Tamoxifen.
c) Spironolactone.
d) Danazol.
Copyrights apply
Fifteen – year old boy, has presented with tender
gynecomastia (3 cm in diameter bilaterally). He is in
early to mid puberty. Which one of the following is first
option in the management?
a)Treatment with an anti-estrogen (Tamoxifen).
b)Treatment with an aromatase inhibitor.
c)Surgery.
d)Reassurance.
• Gynecomastia is the benign proliferation of
glandular breast tissue in males.
• Pubertal Gynecomastia is common from
puberty till age of 25 years.
• It resolves spontaneously without any
intervention.
• It is most likely due to aromatization of
testosterone into estrogen.
A thorough history should be obtained that
addresses the following:
• Age of onset and duration of the condition
• Any recent changes in nipple size and any pain
or discharge from the nipples
• History of mumps, testicular trauma, alcohol
use, or drug use
• Family history of Gynecomastia
• History of sexual dysfunction, infertility, or
hypogonadism
Physical examination should include the following:
• Thorough examination of the breasts, with
attention to size and consistency
• Assessment for any nipple discharge or axillary
lymphadenopathy
• Testing to differentiate between true
Gynecomastia and pseudogynecomastia
• Assessment of glandular tissue
• Examination of the testicles, with attention to
size and consistency, as well as nodules or
asymmetry
• Observation of any signs of feminization
• Checking for any stigmata of chronic liver disease,
thyroid disease, or renal disease
Approach Considerations
• Generally, no treatment is required for physiologic
Gynecomastia.
• Pubertal Gynecomastia resolves spontaneously within
several weeks to 3 years in approximately 90% of patients.
• Breasts greater than 4 cm in diameter may not completely
regress.
• If hypogonadism (primary or secondary) is the cause of
Gynecomastia, parenteral or transdermal testosterone
replacement therapy is instituted.
• However, testosterone does have the potential to exacerbate
Gynecomastia through the aromatization of the exogenous
hormone into estradiol.
• idiopathic Gynecomastia or with residual Gynecomastia after
treatment of the primary cause, medical or surgical
treatment may be considered.
Pharmacologic Therapy
• With the administration of clomiphene, an antiestrogen,
approximately 50% of patients achieve partial reduction in
breast size, and approximately 20% of patients note
complete resolution.
– Adverse effects, while rare, include visual problems, rash, and
nausea.
• Tamoxifen, an estrogen antagonist, is effective for recent-
onset and tender Gynecomastia.
• Up to 80% of patients report partial to complete resolution.
Nausea and epigastric discomfort are the main adverse
effects.
• Other drugs used, albeit less frequently, include
danazol. Danazol, a synthetic derivative of testosterone,
inhibits pituitary secretion of LH and follicle-stimulating
hormone (FSH), which decreases estrogen synthesis from
the testicles.
Sixteen –year- old adolescent, noted to have
firm mass behind the right nipple. The mass is
mobile, non tender. Select the most likely
diagnosis?
a) Lymphoma.
b) Intraductal papilloma.
c) Gynecomastia.
d) Benign cyst.
• Gynecomastia is a benign enlargement of the
male breast (usually bilateral but sometimes
unilateral) resulting from a proliferation of the
glandular component of the breast.
• It is defined clinically by the presence of
rubbery or firm mass extending concentrically
from the nipples.
• Gynecomastia should be differentiated from
pseudo gynecomastia (lipomata), which is
characterized by fat deposition without
glandular proliferation.
Seven - years old boy has presented with vomiting,
constipation, lethargy & excessive urination. No
dysmorphic signs were noted. His serum calcium was
2.9 mmol/l (2.1-2.5), phosphate 0.8 (1.1-1.8), urinary
calcium: creatinine ratio was high, PTH level was100 (6-
15) &vitamin D was 77 (75-125). Which one of the
following is the most likely cause?
a) William’s syndrome.
b) Familial hypocalciuric hypercalcemia.
c) Hypervitaminosis D.
d) Parathyroid adenoma.
Hypercalcemia in children
• It may present with hypotonia, poor feeding,
vomiting, constipation, abdominal pain, lethargy,
polyuria, dehydration, failure to thrive and
seizures.
• In severe cases renal failure, pancreatitis and
reduced consciousness may also occur.
• In adolescents may present with psychiatric
symptoms.
• The causes of hypercalcemia in children can be
classified as PTH‐dependent or PTH‐independent.
• Could be congenital or acquired.
Hypercalcemia in children
• PTH‐independent hypercalcemia:
– hypervitaminosis D.
– granulomatous disorders.
– idiopathic infantile hypercalcemia.
– William's syndrome.
• PTH‐dependent hypercalcemia:
– parathyroid tumors, which may give rise to
primary hyperparathyroidism (PHPT).
– tertiary hyperparathyroidism, which usually arises
in association with chronic renal failure.
– treatment of hypophosphatemic rickets.
• In approximately 85% of cases, primary
hyperparathyroidism is caused by a single adenoma.
• In 15% of cases, multiple glands are involved (i.e.,
either multiple adenomas or hyperplasia).
• Rarely, primary hyperparathyroidism is caused by
parathyroid carcinoma.
• Familial cases can occur as either part of the multiple
endocrine neoplasia syndromes (MEN 1 or MEN 2a),
hyper parathyroid-jaw tumor (HPT-JT) syndrome, or
familial isolated hyperparathyroidism (FIHPT).
• Familial hypocalciuric hypercalcemia and neonatal
severe hyperparathyroidism also belong to this
category.
Otherwise healthy 6-week infant presented with
generalized seizure. She was, somewhat sleepy
with normal examinations. Her glucose was 4.1
mmol/l, sodium 141 mmol/l, calcium 1.5
mmol/l, phosphorus 2.1 mmol/l, magnesium 0.8
mmol/l & ALP 200 IU/l. Which one of the
following is most likely diagnosis?
a) Pseudohypoparathyroidism.
b) Hypoparathyroidism.
c) Vitamin D deficiency.
d) Albright’s hereditary osteodystrophy.
• Hypoparathyroidism may be transient, inherited,
or acquired.
• Transient hypoparathyroidism:
– Preterm and low birth neonates are at
increased risk, and as many as 50% of them
might have a deficient surge in PTH that
results in hypocalcemia.
– It is possible that the functional immaturity is
a manifestation of a delay in development of
the enzymes that convert glandular PTH to
secreted PTH.
• Permanent hypoparathyroidism:
– DiGeorge syndrome (hypoparathyroidism,
absence of thymus gland with T-cell
abnormalities, and cardiac anomalies) is
associated with abnormal development of the
third and fourth pharyngeal pouches from which
the parathyroid derive embryologically and
represents an example of a defect in parathyroid
gland development.
– DiGeorge syndrome and velocardiofacial
syndrome are variants of the chromosome arm
22q11 micro deletion syndrome.
• X-linked recessive hypoparathyroidism has
been associated with parathyroid agenesis
and has been mapped to chromosome arm
Xq26-q27, the location of a putative
developmental gene.
• Familial cases of hypoparathyroidism due to
mutations of the PTH gene located on
chromosome arm 11p15 have been
identified.
• These mutations have been both dominantly
and recessively inherited.
Eleven year old boy, presented with mental sub -
normality, facial dysmorphism (photo), and severe
growth failure. Which diagnostic laboratory investigation,
you are going to order?
a) IGF- 1.
b) GH provocative test.
c) Bone age.
d) Serum calcium & PTH.
• Sanjad Sakati syndrome (SSS) is an autosomal
recessive disorder found exclusively in people of
Arabian origin.
• It is first reported from the Kingdom of Saudi Arabia in
1988 as a newly described syndrome mainly from the
Middle East and the Arabian Gulf countries.
• Children affected with this condition are born small for
gestational age and present with hypocalcemic tetany
or seizures due to hypoparathyroidism at an early
stage in their lives. They have typical physical features,
namely; long narrow face, deep set small eyes, beaked
nose, large floppy ears, micrognathia, severe failure to
grow both intrauterine and extra uterine and mild to
moderate mental retardation.
seventeen - year old adolescent, presented with
hirsutism, irregular menses and acne. Which of
the following supports the diagnosis of PCOS?
a) High FSH level.
b) LH: FSH ratio of 3:1.
c) LH: FSH ratio of 1:1.
d) High estradiol level.
Definitions
• Primary amenorrhea: Lack of menses by 15 years of
age or three years after the onset of breast
development.
• Secondary amenorrhea: More than 90 days without a
menstrual period, after previously menstruating.
• Irregular menstruation cycle: defined as if it is less than
24 days or more than 38 days.
• Irregular menstruation is common in adolescence.
• A regular menstrual cycle can be set within a year of
first menses.
PCOS
• Most expert groups use Rotterdam criteria to make the
diagnosis of PCOS.
• Two out of three of the following criteria are required to
make the diagnosis:
– Oligo- and/or anovulation.
– Clinical and/or biochemical signs of hyperandrogenism.
– Polycystic ovaries (by ultrasound).
• The diagnosis of PCOS is confirmed once other conditions
with features similar to PCOS have been excluded, such as:
– non-classic congenital adrenal hyperplasia, hypothyroidism,
hyperprolactinemia, androgen-secreting ovarian and adrenal
tumors.
• Polycystic ovary syndrome (PCOS), frequently
becomes manifest during adolescence, and is
primarily characterized by ovulatory dysfunction
and hyperandrogenism.
• The syndrome is heterogeneous clinically and
biochemically.
• The diagnosis of PCOS has lifelong implications
with increased risk for metabolic syndrome, type
2 diabetes mellitus, and possibly cardiovascular
disease and endometrial carcinoma.
• PCOS should be considered in any adolescent girl
presenting with a chief complaint of hirsutism,
treatment-resistant acne, menstrual irregularity,
acanthosis nigricans, and/or obesity
Fifteen –year – old, obese adolescent female who has
hirsutism & acne. Which one of the following, is most
likely diagnosis?
a) Cushing syndrome.
b) Polycystic ovarian syndrome.
c) Virilizing adrenal tumor.
d) Non-classical CAH.
Hirsutism versus virilization
• Virilization includes clitoromegaly, male‐pattern baldness,
deepening of the voice, and increased muscle mass in
addition to the clinical features of hirsutism and chronic
anovulation
• The magnitude or amount of excessive hair growth can be
approximated by the Ferriman‐Gallwey score
• With this method, the amount of hair growth in nine
androgen‐dependent areas is compared to a standard chart
(grades 1 to 4) from which a score is derived.
• Grade 1 indicates minimal terminal hair growth and grade 4
indicates dense terminal hair growth.
• Scores greater than 8 are considered to indicate hirsutism
Ferriman‐Gallwey score
Hypertrichosis / Hirsutism
• Generalized hypertrichosis is a common adverse side effect of
several medications.
• Starvation, whether due to malnutrition or anorexia nervosa,
and hypothyroidism can cause acquired hypertrichosis.
• Several genetic disorders can be associated with excessive
generalized hair growth.
• Leprechaunism is characterized by hypertrichosis and insulin
resistance due to mutations in the insulin receptor gene
• Hirsutism is defined as excessive growth of coarse terminal
hairs in androgen‐dependent areas such as the face, upper
chest, abdomen, and back
• Hypertrichosis is excessive hair growth in both
androgen‐dependent & androgen‐independent areas.
Thirteen year old adolescent presented with
hirsutism. Which one of the following
medications is therapeutic?
a) Thiazide diuretic.
b) Oral contraceptives.
c) Sildenafil.
d) Minoxidil.
• Contraceptives can treat hirsutism in 2/3 of
cases.
• Direct suppression of ovarian steroid
production and increase hepatic binding
globulin production, which binds circulating
androgens which lead to decrease free
androgens.
Eighteen months old boy was referred for further
assessment of his increasingly bowing of legs. His
parents are first degree cousins. He has been on
vitamin D3 therapy for last 6 months, dose of 3000
unit/day with good compliance. His serum calcium was
1.37mmol/L, phosphate 0.13mmol/L , alkaline
phosphatase 805IU/L, PTH 100 (6-15) pmol/L. Which
one of the following is a diagnostic investigation?
a) Do wrist x- ray to confirm active rickets.
b) Do 25- hydroxy vitamin D metabolite.
c) Do both 25 &1,25- di hydroxy vitamin d metabolites.
d) Repeat bone profile next day after correction of
hypocalcemia.
HRVD type 2
HRVD type 2
• It is a very rare form of rickets.
• Reported in approximately 100 cases only so far
worldwide.
• HRVD is an autosomal recessive disorder.
• It is associated with end–organ resistance to 1,25-
dihydroxyvitamin D (1,25[OH]2D).
• Usually caused by mutations in the gene encoding
the vitamin D receptor.
• The defect in the receptor interferes with the
function of the hormone-receptor complex, thereby
preventing 1,25(OH)2D action.
HRVD type 2
• Affected children usually appear normal at birth.
• Develop rickets within the first two years of life.
• Alopecia and ectodermal anomalies resulting
from the lack of vitamin D receptor activity within
keratinocytes develops in approximately two-
thirds of cases and is a marker of disease severity
(HRVD type 2A).
• Other patients without alopecia or other
ectodermal anomalies (HRVD type 2B).
HRVD type 2A
Twelve –year old boy, who has presented with height
below 3rd.%, with growth velocity of 3 cm/ year. Both
parents were of average height (MPH was 168 cm
between 10th-25th%). On examination, Tanner staging
was B3 PH 2. Among which one of the following is most
appropriate approach?
a) Observe & follow up in next six months.
b) Re-assure the family, shortly she will catch- up.
c) Do bone age only.
d) Do full hormonal assessments workup.
Short stature
• Is defined as height that is 2 standard deviations
(SD) or more below the mean height for children of
that sex and chronologic age in a given population.
• The clinical significance of short stature depends on
many factors, including genetic potential and height
velocity .
• Determination of the child's growth (HV) is essential
component of the evaluation for short stature.
– Age two to four years – HV less than 5.5 cm/year
– Age four to six years – HV less than 5 cm/year
– Age six years to puberty: HV less than 4 cm/year
for boys & less than 4.5 cm/year for girls.
• Target height female = ((Height of father - 13) + Height of
mother) / 2 ±8.5 cm.
• Target height male = ((Height of mother + 13) + Height of
father) / 2 ±8.5 cm.
• Family history of growth & pubertal onset.
• Review of systems for features suggestive of
gastrointestinal, pulmonary, immunologic, or other
systemic disease.
• Dysmorphic features, especially webbed neck, cubitus
valgus, and absent puberty in girls (suggests Turner
syndrome), or disproportionate short stature (i.e., short
limbs compared with trunk) suggestive of skeletal
dysplasia.
• Laboratory evaluation for a child with short stature
depends on clinical evaluation.
• The bone age used to the estimate for the child's
adult height and in the evaluation for possible causes
of short stature.
• Children with pathological short stature should be
evaluated with CBC, ESR, tTG IgA, creatinine,
electrolytes, TSH, free thyroxine, IGF-1, and IGFBP-3.
• A karyotype to evaluate for Turner syndrome.
• Skeletal survey for children suspected skeletal
dysplasia.
• Clonidine, arginine and glucagon are common choices
in children.
• Hypothyroidism should be excluded first by performing thyroid
function tests.
• GH stimulation testing is not necessary for selected patients in
whom other clinical criteria are sufficient to make the
diagnosis of GHD, including:
– Those with a known pituitary abnormality (congenital
anomaly, tumor, or irradiation).
– Those with known deficiency of at least one other pituitary
hormone, with marked growth failure.
• If GHD is confirmed, MRI of the hypothalamic-pituitary area is
recommended to:
– rule out tumors.
– investigate for structural causes of GHD.
– to evaluate the severity & prognosis of the deficiency.
Six- year old boy, known case of growth
hormone deficiency referred from orthopedic
clinic for second opinion, whether or not to
continue or not, as he has thoraco-lumbar
scoliosis. Which one of the following is the best
opinion?
a) No worries, and continue GH safely.
b) Stop GH therapy immediately.
c) Decrease the dose and continue GH.
d) Continue with close observation, hand with hand
with orthopedic surgeon.
Scoliosis & GH therapy
• Scoliosis related to rapid growth that occurs
with therapy and is not a direct effect of the
growth hormone.
• Patients with scoliosis who are treated with
growth hormone should have their scoliosis
monitored during therapy.
• No clear statement to stop GH therapy, rather
close monitoring during therapy.
Contraindications of GH therapy
• GH should not be used for pediatric treatment if
patient's growth plates (epiphyses) are closed.
• Active proliferative or severe non-proliferative
diabetic retinopathy.
• GH should not be used in patients with any
evidence of any tumor.
• GH should not be used in patients pre-existing or
active malignancy.
Contraindications of GH therapy
• GH should not be used in patients with
complication due to open heart or abdominal
surgery and with acute respiratory failure.
• GH is contraindicated in severely obese
patients or with respiratory impairments.
• GH is contraindicated in patients with Prader-
Willi syndrome who are severely obese or have
severe respiratory impairment.
Which one of the following is not proven adverse
effect of GH replacement therapy?
a) Carpal tunnel syndrome.
b) Arthralgia and myalgia.
c) Benign intracranial hypertension.
d) Increase incidence of brain tumor.
Increase incidence of brain tumor is never
documented & is not proven adverse effect
of growth hormone therapy.
GH side effects
• Allergic reaction.
• Ongoing injection site discomfort.
• Pain in wrist (carpal tunnel).
• Curvature of the spine (scoliosis).
• Joint pain.
• Puffy hands and/or feet (caused by fluid
retention).
• Changes in vision, headache, nausea.
• Hip or knee pains.
• Limping.
Eight-year-old girl was referred because of tall
stature. She has on & off headache. On
examination there were no dysmorphic features.
Her height is just above the 99th % and her
parents’ MPH between 50th to 75th %. Tanner
staging was: B2, PH 3, no menarche. Bone age
was 12.4 years. Her final height prediction is 188
cm. Which one of the following statement is
most appropriate?
a) Most likely familial need to observe growth
velocity.
b) Need to do basal & stimulated GH test.
c) Need to do IGF -1.
d) Need to do oral glucose tolerance test for GH
suppression.
Tall stature
• Children and adolescents with a height in excess of two
standard deviations above the mean height for age are
considered tall.
• Obesity can also lead to tall stature in childhood. However, in
such children, puberty usually occurs early, with a resulting
normal final adult height.
• Other causes of tall stature can include excessive secretion of
growth hormone or sex hormones
• In the presence of excessive sex hormones, there will be early
growth acceleration, initially leading to tall stature,
subsequently, there will usually be early epiphyseal closure,
resulting in short stature after puberty.
• An increased height velocity and advanced bone age suggest
excessive production of sex hormones, growth hormone, or,
occasionally, thyroid hormone.
• Familial tall stature is a normal variant and is associated with
normal height velocity and tall parents
• Long extremities are associated with homocystinuria, Marfan’s
syndrome, and Y chromosome disorders, whereas normal
extremities are found in estrogen insensitivity and the other
syndromes associated with tall stature.
• More girls than boys seek medical attention for tallness
because of the perceived negative social consequences of the
condition.
• In the rare instances in which treatment is indicated, relatively
high doses of sex steroids are introduced early to advance
osseous maturation.
Six month old male infant presented with
failure to thrive, constipation. His mother is
complaining of too many diaper change and
urine is leaking out of diapers most of the
time. On examination he is having moderate to
severe dehydration. His initial sodium is high
175 mmol/l., very low urine osmolality. Which
one of the following is least common cause in
the differential diagnosis of this infant?
a) Langerhans cell histiocytosis.
b) X-linked recessive nephrogenic DI.
c) DIDMOAD syndrome.
d) Psychological polydipsia.
Central DI
• The three most common causes of cranial
diabetes insipidus are:
– brain tumor that damages the hypothalamus or
pituitary gland, which accounts for 25% of cases
– severe head injury that damages the
hypothalamus or pituitary gland, which accounts
for 16% of cases
– complications that occur during brain surgery,
which account for 20% of cases
Central DI
• Less common causes of cranial diabetic insipidus
include:
– cancers that spread from another part of the body,
such as the lungs or the bone marrow, to the brain
– Wolfram syndrome, a rare genetic disorder that also
causes sight and vision loss
– brain damage caused by a sudden loss of oxygen,
which can occur during a stroke or drowning
– infections, such as meningitis and encephalitis, that
can damage the brain
Which one of the following is first line treatment of
acute hypercalcemia?
a) Calcitonin.
b) Diuretics.
c) Intravenous hydration.
d) Bisphosphonate therapy.
Which one of the following is commonest cause of 46
XX DSD?
a) Partial AIS.
b) CAH (21- OH deficiency).
c) Virilizing ovarian or adrenal tumors.
d) Placental aromatase enzyme deficiency.
3/23/2020 158
Which one of the following is commonest cause
of 46 XY DSD?
a) Testicular Hypoplasia.
b) Partial AIS.
c) Testosterone biosynthesis defects.
d) 5  - Reductase deficiency.
Three years old boy presented with goiter, short stature,
deafness and symptoms suggestive of mild
hypothyroidism. On examination, is having normal
mentality, diffuse goiter, deaf and mute with normal
CNS examination apart from sluggish reflexes. His bone
age is retarded and has raised level of circulating TSH,
fT4 and fT3. Which of the following is most likely
diagnosis?
a) Generalized resistance to thyroid hormone (GRTH)
b) Pituitary resistance to thyroid hormone (PRTH)
c) Pendred's syndrome
d) TSH secreting Adenoma
Resistance to thyroid hormone (RTH)
• Is usually dominantly inherited
• Characterized by elevated fT3 &fT4 and failure
to suppress TSH secretion.
• Variable refractoriness to hormone action in
peripheral tissues.
• Two major forms:
– asymptomatic individuals with generalized
resistance (GRTH).
– patients with thyrotoxicosis features, suggesting
predominant pituitary resistance (PRTH).
• Recognized features of RTH include failure to thrive,
growth retardation , ADHD in childhood, goiter and
thyrotoxic cardiac symptoms in adults.
• The most common cause of the syndrome are
mutations of the β (beta) form (THRB gene) of the
thyroid hormone receptor, of which over 100 different
mutations have been documented.
• Thyroid hormone resistance syndrome is rare,
incidence is variously quoted as 1 in 50,000 or 1 in
40,000 live births.
Resistance to thyroid hormone (RTH)
• The characteristic blood test results for this
disorder can also be found in other disorders
(for example TSH-oma (pituitary adenoma), or
other pituitary disorders).
• The diagnosis may involve identifying
mutation of the thyroid receptor, which is
present in approximately 85% of cases.
Which of the following is a cause of congenital
nephrogenic diabetes insipidus?
a) Hypercalcemia.
b) Hypokalemia .
c) Post obstructive uropathy.
d) Vasopressine receptor gene mutation .
Vasopressin receptor gene mutation is the cause
of primary nephrogenic DI not the acquired
form.
Congenital nephrogenic diabetes insipidus
• Two genetic have been identified that cause
nephrogenic diabetes insipidus present at
birth.
– The first is known as the AVRP2 gene mutation
and is responsible for 90% of all cases of
congenital diabetes insipidus (although it is still
very rare, occurring in an estimated 1 in every
250,000 births).
– The remaining 10% of cases of congenital
nephrogenic diabetes insipidus are caused by the
AQP2 gene mutation, which can affect both males
and females
Acquired nephrogenic diabetes insipidus
• The most common cause is the medication
lithium.
• Lithium is widely used to treat bipolar disorder
• over half of all people on long-term lithium
therapy will develop nephrogenic diabetes
insipidus.
• Stopping treatment with lithium will restore
normal kidney function, though in many cases the
damage to the kidneys is permanent.
Acquired nephrogenic diabetes insipidus
• Other causes include:
– hypercalcemia
– hypokalemia
– pyelonephritis
– ureteral obstruction
Which one of the following disorders of growth
is characterized by normal body proportion ?
a) Achondroplasia.
b) Morquio’s.
c) Congenital hypothyroidism.
d) Hypopituitarism.
GOOD LUCK

Más contenido relacionado

La actualidad más candente

Delayed puberty , etiology , diagnostic approach
Delayed puberty , etiology , diagnostic approach Delayed puberty , etiology , diagnostic approach
Delayed puberty , etiology , diagnostic approach Aftab Siddiqui
 
Basic approach on short stature in children
Basic approach on short stature in childrenBasic approach on short stature in children
Basic approach on short stature in childrenAzad Haleem
 
Pediatric endocrine revesion (part 1)
Pediatric endocrine revesion (part 1) Pediatric endocrine revesion (part 1)
Pediatric endocrine revesion (part 1) Abdulmoein AlAgha
 
Pediatric endocrinology review - part 3
Pediatric endocrinology review -  part 3Pediatric endocrinology review -  part 3
Pediatric endocrinology review - part 3Abdulmoein AlAgha
 
Approach to dsd siddarth mahajan
Approach to dsd  siddarth mahajanApproach to dsd  siddarth mahajan
Approach to dsd siddarth mahajanDr Praman Kushwah
 
approach to short stature
approach to short statureapproach to short stature
approach to short statureRatnakar Vallem
 
Neonatal hypocalcemia
Neonatal hypocalcemiaNeonatal hypocalcemia
Neonatal hypocalcemiaMostafa Galal
 
Hypocalcemia in paediatrics
Hypocalcemia in paediatricsHypocalcemia in paediatrics
Hypocalcemia in paediatricsmissmarimo
 
Congenital Hypothyrodism
Congenital HypothyrodismCongenital Hypothyrodism
Congenital HypothyrodismLifecare Centre
 
Approach to a child with Rickets
Approach to a child with Rickets Approach to a child with Rickets
Approach to a child with Rickets Nassr ALBarhi
 
Hyperparathyroidism in children
Hyperparathyroidism in childrenHyperparathyroidism in children
Hyperparathyroidism in childrenJoyce Mwatonoka
 
Practical pediatric quiz - Kaun Banega Winner
Practical pediatric quiz - Kaun Banega WinnerPractical pediatric quiz - Kaun Banega Winner
Practical pediatric quiz - Kaun Banega WinnerGaurav Gupta
 

La actualidad más candente (20)

Precocious puberty
Precocious puberty   Precocious puberty
Precocious puberty
 
Growth Hormone Deficiency in Children
Growth Hormone Deficiency in ChildrenGrowth Hormone Deficiency in Children
Growth Hormone Deficiency in Children
 
Delayed puberty , etiology , diagnostic approach
Delayed puberty , etiology , diagnostic approach Delayed puberty , etiology , diagnostic approach
Delayed puberty , etiology , diagnostic approach
 
Basic approach on short stature in children
Basic approach on short stature in childrenBasic approach on short stature in children
Basic approach on short stature in children
 
Precocious puberty
Precocious pubertyPrecocious puberty
Precocious puberty
 
Pediatric endocrine revesion (part 1)
Pediatric endocrine revesion (part 1) Pediatric endocrine revesion (part 1)
Pediatric endocrine revesion (part 1)
 
Pediatric endocrinology review - part 3
Pediatric endocrinology review -  part 3Pediatric endocrinology review -  part 3
Pediatric endocrinology review - part 3
 
Approach to dsd siddarth mahajan
Approach to dsd  siddarth mahajanApproach to dsd  siddarth mahajan
Approach to dsd siddarth mahajan
 
Congenital adrenal hyperplasia
Congenital adrenal hyperplasiaCongenital adrenal hyperplasia
Congenital adrenal hyperplasia
 
approach to short stature
approach to short statureapproach to short stature
approach to short stature
 
Neonatal hypocalcemia
Neonatal hypocalcemiaNeonatal hypocalcemia
Neonatal hypocalcemia
 
Hypocalcemia in paediatrics
Hypocalcemia in paediatricsHypocalcemia in paediatrics
Hypocalcemia in paediatrics
 
Congenital Hypothyrodism
Congenital HypothyrodismCongenital Hypothyrodism
Congenital Hypothyrodism
 
Delayed puberty in children
Delayed puberty in childrenDelayed puberty in children
Delayed puberty in children
 
Pcd presentation mpls
Pcd presentation mplsPcd presentation mpls
Pcd presentation mpls
 
Approach to a child with Rickets
Approach to a child with Rickets Approach to a child with Rickets
Approach to a child with Rickets
 
Conn’s syndrome
Conn’s syndromeConn’s syndrome
Conn’s syndrome
 
Primary hyperaldosteronism
Primary hyperaldosteronismPrimary hyperaldosteronism
Primary hyperaldosteronism
 
Hyperparathyroidism in children
Hyperparathyroidism in childrenHyperparathyroidism in children
Hyperparathyroidism in children
 
Practical pediatric quiz - Kaun Banega Winner
Practical pediatric quiz - Kaun Banega WinnerPractical pediatric quiz - Kaun Banega Winner
Practical pediatric quiz - Kaun Banega Winner
 

Similar a Pediatric endocrinology review questions

Dar_Shahid_Yousuf_Iws.pptx
Dar_Shahid_Yousuf_Iws.pptxDar_Shahid_Yousuf_Iws.pptx
Dar_Shahid_Yousuf_Iws.pptxUmairFirdous
 
PEDI GU REVIEW-External Genitalia
PEDI GU REVIEW-External GenitaliaPEDI GU REVIEW-External Genitalia
PEDI GU REVIEW-External GenitaliaGeorge Chiang
 
Genetic counselling 7 march13-Dr.Gourav
Genetic counselling 7 march13-Dr.GouravGenetic counselling 7 march13-Dr.Gourav
Genetic counselling 7 march13-Dr.GouravGourav Thakre
 
Precocious puberty
Precocious pubertyPrecocious puberty
Precocious pubertyDr Slayer
 
Unit 1_ Genetic Disorders, Part 2, Educational Platform.ppt
Unit 1_ Genetic Disorders, Part 2, Educational Platform.pptUnit 1_ Genetic Disorders, Part 2, Educational Platform.ppt
Unit 1_ Genetic Disorders, Part 2, Educational Platform.pptuk581147
 
Down syndrome
Down syndromeDown syndrome
Down syndromeJihajie
 
infertility-140601034142-phpapp02 2.pdf
infertility-140601034142-phpapp02 2.pdfinfertility-140601034142-phpapp02 2.pdf
infertility-140601034142-phpapp02 2.pdfFadilaLawal
 
ADOLESCENT HEALTH AND DISORDERS OF PUBERTY.pptx
ADOLESCENT HEALTH AND DISORDERS OF PUBERTY.pptxADOLESCENT HEALTH AND DISORDERS OF PUBERTY.pptx
ADOLESCENT HEALTH AND DISORDERS OF PUBERTY.pptxneha102811
 
Pediatric endocrinology review part 2
Pediatric endocrinology review  part 2 Pediatric endocrinology review  part 2
Pediatric endocrinology review part 2 Abdulmoein AlAgha
 
Infant of a diabetic mother
Infant of a diabetic mother Infant of a diabetic mother
Infant of a diabetic mother Yassin Alsaleh
 

Similar a Pediatric endocrinology review questions (20)

Dar_Shahid_Yousuf_Iws.pptx
Dar_Shahid_Yousuf_Iws.pptxDar_Shahid_Yousuf_Iws.pptx
Dar_Shahid_Yousuf_Iws.pptx
 
Genetic counseling
Genetic counselingGenetic counseling
Genetic counseling
 
PEDI GU REVIEW-External Genitalia
PEDI GU REVIEW-External GenitaliaPEDI GU REVIEW-External Genitalia
PEDI GU REVIEW-External Genitalia
 
Down syndrom
Down syndromDown syndrom
Down syndrom
 
Down syndrom
Down syndromDown syndrom
Down syndrom
 
Genetic counselling 7 march13-Dr.Gourav
Genetic counselling 7 march13-Dr.GouravGenetic counselling 7 march13-Dr.Gourav
Genetic counselling 7 march13-Dr.Gourav
 
Precocious puberty
Precocious pubertyPrecocious puberty
Precocious puberty
 
Down Syndrome
Down SyndromeDown Syndrome
Down Syndrome
 
Unit 1_ Genetic Disorders, Part 2, Educational Platform.ppt
Unit 1_ Genetic Disorders, Part 2, Educational Platform.pptUnit 1_ Genetic Disorders, Part 2, Educational Platform.ppt
Unit 1_ Genetic Disorders, Part 2, Educational Platform.ppt
 
Metabolic screening in newborn
Metabolic screening in newborn   Metabolic screening in newborn
Metabolic screening in newborn
 
Down syndrome
Down syndromeDown syndrome
Down syndrome
 
Turner's syndrome
Turner's syndromeTurner's syndrome
Turner's syndrome
 
infertility-140601034142-phpapp02 2.pdf
infertility-140601034142-phpapp02 2.pdfinfertility-140601034142-phpapp02 2.pdf
infertility-140601034142-phpapp02 2.pdf
 
Infertility
InfertilityInfertility
Infertility
 
ADOLESCENT HEALTH AND DISORDERS OF PUBERTY.pptx
ADOLESCENT HEALTH AND DISORDERS OF PUBERTY.pptxADOLESCENT HEALTH AND DISORDERS OF PUBERTY.pptx
ADOLESCENT HEALTH AND DISORDERS OF PUBERTY.pptx
 
Pediatric endocrinology review part 2
Pediatric endocrinology review  part 2 Pediatric endocrinology review  part 2
Pediatric endocrinology review part 2
 
Down's syndrome
Down's syndromeDown's syndrome
Down's syndrome
 
Infant of a diabetic mother
Infant of a diabetic mother Infant of a diabetic mother
Infant of a diabetic mother
 
Chromosomal abnormalities
Chromosomal abnormalitiesChromosomal abnormalities
Chromosomal abnormalities
 
Pediatric radiology
Pediatric radiologyPediatric radiology
Pediatric radiology
 

Más de Abdulmoein AlAgha

Type 2 DM in children & adolescents management overview
Type 2 DM in children & adolescents management overviewType 2 DM in children & adolescents management overview
Type 2 DM in children & adolescents management overviewAbdulmoein AlAgha
 
Thyroid disorders in children
Thyroid disorders in childrenThyroid disorders in children
Thyroid disorders in childrenAbdulmoein AlAgha
 
Metabolic bone disease with focus on hypophosphatasia
Metabolic bone disease with focus on hypophosphatasiaMetabolic bone disease with focus on hypophosphatasia
Metabolic bone disease with focus on hypophosphatasiaAbdulmoein AlAgha
 
The role of nutrition in children growth &amp; health
The role of nutrition in children growth &amp; healthThe role of nutrition in children growth &amp; health
The role of nutrition in children growth &amp; healthAbdulmoein AlAgha
 
Diabetes technology &patient self care
Diabetes technology &patient self careDiabetes technology &patient self care
Diabetes technology &patient self careAbdulmoein AlAgha
 
Vitamin d deficiency &amp; rickets
Vitamin d deficiency &amp; ricketsVitamin d deficiency &amp; rickets
Vitamin d deficiency &amp; ricketsAbdulmoein AlAgha
 
Pediatric growth hormone deficiency
Pediatric growth hormone deficiencyPediatric growth hormone deficiency
Pediatric growth hormone deficiencyAbdulmoein AlAgha
 
Types of insulin &amp; correction of hyperglycemia
Types of insulin &amp; correction of hyperglycemiaTypes of insulin &amp; correction of hyperglycemia
Types of insulin &amp; correction of hyperglycemiaAbdulmoein AlAgha
 
Vitamin d deficiency in children
Vitamin d deficiency in childrenVitamin d deficiency in children
Vitamin d deficiency in childrenAbdulmoein AlAgha
 
Various types of diabetes in children
Various types of diabetes in childrenVarious types of diabetes in children
Various types of diabetes in childrenAbdulmoein AlAgha
 
Fating Ramadan &amp; type 1 diabetes
Fating Ramadan &amp; type 1 diabetesFating Ramadan &amp; type 1 diabetes
Fating Ramadan &amp; type 1 diabetesAbdulmoein AlAgha
 
Short stature definition and approach
Short stature definition and approachShort stature definition and approach
Short stature definition and approachAbdulmoein AlAgha
 
Puberty normal and precocious
Puberty normal and precociousPuberty normal and precocious
Puberty normal and precociousAbdulmoein AlAgha
 
Approach to hypoglycemia in infants and children
Approach to hypoglycemia in infants and childrenApproach to hypoglycemia in infants and children
Approach to hypoglycemia in infants and childrenAbdulmoein AlAgha
 

Más de Abdulmoein AlAgha (20)

Type 2 DM in children & adolescents management overview
Type 2 DM in children & adolescents management overviewType 2 DM in children & adolescents management overview
Type 2 DM in children & adolescents management overview
 
Thyroid disorders in children
Thyroid disorders in childrenThyroid disorders in children
Thyroid disorders in children
 
Growth hormone testing
Growth hormone testingGrowth hormone testing
Growth hormone testing
 
Metabolic bone disease with focus on hypophosphatasia
Metabolic bone disease with focus on hypophosphatasiaMetabolic bone disease with focus on hypophosphatasia
Metabolic bone disease with focus on hypophosphatasia
 
The role of nutrition in children growth &amp; health
The role of nutrition in children growth &amp; healthThe role of nutrition in children growth &amp; health
The role of nutrition in children growth &amp; health
 
Diabetes technology &patient self care
Diabetes technology &patient self careDiabetes technology &patient self care
Diabetes technology &patient self care
 
Vitamin d deficiency &amp; rickets
Vitamin d deficiency &amp; ricketsVitamin d deficiency &amp; rickets
Vitamin d deficiency &amp; rickets
 
Pediatric growth hormone deficiency
Pediatric growth hormone deficiencyPediatric growth hormone deficiency
Pediatric growth hormone deficiency
 
Types of insulin &amp; correction of hyperglycemia
Types of insulin &amp; correction of hyperglycemiaTypes of insulin &amp; correction of hyperglycemia
Types of insulin &amp; correction of hyperglycemia
 
Vitamin d deficiency in children
Vitamin d deficiency in childrenVitamin d deficiency in children
Vitamin d deficiency in children
 
Various types of diabetes in children
Various types of diabetes in childrenVarious types of diabetes in children
Various types of diabetes in children
 
Growth hormone therapy
Growth hormone therapyGrowth hormone therapy
Growth hormone therapy
 
Fating Ramadan &amp; type 1 diabetes
Fating Ramadan &amp; type 1 diabetesFating Ramadan &amp; type 1 diabetes
Fating Ramadan &amp; type 1 diabetes
 
Diabetes+ketoacidosis
Diabetes+ketoacidosisDiabetes+ketoacidosis
Diabetes+ketoacidosis
 
Degludec presentation
Degludec presentation Degludec presentation
Degludec presentation
 
Short stature definition and approach
Short stature definition and approachShort stature definition and approach
Short stature definition and approach
 
type 1 diabetes in children
type 1 diabetes in childrentype 1 diabetes in children
type 1 diabetes in children
 
Safety of GH therapy
Safety of GH therapySafety of GH therapy
Safety of GH therapy
 
Puberty normal and precocious
Puberty normal and precociousPuberty normal and precocious
Puberty normal and precocious
 
Approach to hypoglycemia in infants and children
Approach to hypoglycemia in infants and childrenApproach to hypoglycemia in infants and children
Approach to hypoglycemia in infants and children
 

Último

Presentation for Bella Mahl 2024-03-28-24-MW-Overview-Bella.pptx
Presentation for Bella Mahl 2024-03-28-24-MW-Overview-Bella.pptxPresentation for Bella Mahl 2024-03-28-24-MW-Overview-Bella.pptx
Presentation for Bella Mahl 2024-03-28-24-MW-Overview-Bella.pptxpdamico1
 
Giftedness: Understanding Everyday Neurobiology for Self-Knowledge
Giftedness: Understanding Everyday Neurobiology for Self-KnowledgeGiftedness: Understanding Everyday Neurobiology for Self-Knowledge
Giftedness: Understanding Everyday Neurobiology for Self-Knowledgeassessoriafabianodea
 
Study on the Impact of FOCUS-PDCA Management Model on the Disinfection Qualit...
Study on the Impact of FOCUS-PDCA Management Model on the Disinfection Qualit...Study on the Impact of FOCUS-PDCA Management Model on the Disinfection Qualit...
Study on the Impact of FOCUS-PDCA Management Model on the Disinfection Qualit...MehranMouzam
 
PERFECT BUT PAINFUL TKR -ROLE OF SYNOVECTOMY.pptx
PERFECT BUT PAINFUL TKR -ROLE OF SYNOVECTOMY.pptxPERFECT BUT PAINFUL TKR -ROLE OF SYNOVECTOMY.pptx
PERFECT BUT PAINFUL TKR -ROLE OF SYNOVECTOMY.pptxdrashraf369
 
Wessex Health Partners Wessex Integrated Care, Population Health, Research & ...
Wessex Health Partners Wessex Integrated Care, Population Health, Research & ...Wessex Health Partners Wessex Integrated Care, Population Health, Research & ...
Wessex Health Partners Wessex Integrated Care, Population Health, Research & ...Wessex Health Partners
 
Nutrition of OCD for my Nutritional Neuroscience Class
Nutrition of OCD for my Nutritional Neuroscience ClassNutrition of OCD for my Nutritional Neuroscience Class
Nutrition of OCD for my Nutritional Neuroscience Classmanuelazg2001
 
MedDRA-A-Comprehensive-Guide-to-Standardized-Medical-Terminology.pdf
MedDRA-A-Comprehensive-Guide-to-Standardized-Medical-Terminology.pdfMedDRA-A-Comprehensive-Guide-to-Standardized-Medical-Terminology.pdf
MedDRA-A-Comprehensive-Guide-to-Standardized-Medical-Terminology.pdfSasikiranMarri
 
Case Report Peripartum Cardiomyopathy.pptx
Case Report Peripartum Cardiomyopathy.pptxCase Report Peripartum Cardiomyopathy.pptx
Case Report Peripartum Cardiomyopathy.pptxNiranjan Chavan
 
CEHPALOSPORINS.pptx By Harshvardhan Dev Bhoomi Uttarakhand University
CEHPALOSPORINS.pptx By Harshvardhan Dev Bhoomi Uttarakhand UniversityCEHPALOSPORINS.pptx By Harshvardhan Dev Bhoomi Uttarakhand University
CEHPALOSPORINS.pptx By Harshvardhan Dev Bhoomi Uttarakhand UniversityHarshChauhan475104
 
LUNG TUMORS AND ITS CLASSIFICATIONS.pdf
LUNG TUMORS AND ITS  CLASSIFICATIONS.pdfLUNG TUMORS AND ITS  CLASSIFICATIONS.pdf
LUNG TUMORS AND ITS CLASSIFICATIONS.pdfDolisha Warbi
 
SGK HÓA SINH NĂNG LƯỢNG SINH HỌC 2006.pdf
SGK HÓA SINH NĂNG LƯỢNG SINH HỌC 2006.pdfSGK HÓA SINH NĂNG LƯỢNG SINH HỌC 2006.pdf
SGK HÓA SINH NĂNG LƯỢNG SINH HỌC 2006.pdfHongBiThi1
 
PULMONARY EDEMA AND ITS MANAGEMENT.pdf
PULMONARY EDEMA AND  ITS  MANAGEMENT.pdfPULMONARY EDEMA AND  ITS  MANAGEMENT.pdf
PULMONARY EDEMA AND ITS MANAGEMENT.pdfDolisha Warbi
 
Statistical modeling in pharmaceutical research and development.
Statistical modeling in pharmaceutical research and development.Statistical modeling in pharmaceutical research and development.
Statistical modeling in pharmaceutical research and development.ANJALI
 
Big Data Analysis Suggests COVID Vaccination Increases Excess Mortality Of ...
Big Data Analysis Suggests COVID  Vaccination Increases Excess Mortality Of  ...Big Data Analysis Suggests COVID  Vaccination Increases Excess Mortality Of  ...
Big Data Analysis Suggests COVID Vaccination Increases Excess Mortality Of ...sdateam0
 
COVID-19 (NOVEL CORONA VIRUS DISEASE PANDEMIC ).pptx
COVID-19  (NOVEL CORONA  VIRUS DISEASE PANDEMIC ).pptxCOVID-19  (NOVEL CORONA  VIRUS DISEASE PANDEMIC ).pptx
COVID-19 (NOVEL CORONA VIRUS DISEASE PANDEMIC ).pptxBibekananda shah
 
Radiation Dosimetry Parameters and Isodose Curves.pptx
Radiation Dosimetry Parameters and Isodose Curves.pptxRadiation Dosimetry Parameters and Isodose Curves.pptx
Radiation Dosimetry Parameters and Isodose Curves.pptxDr. Dheeraj Kumar
 
PNEUMOTHORAX AND ITS MANAGEMENTS.pdf
PNEUMOTHORAX   AND  ITS  MANAGEMENTS.pdfPNEUMOTHORAX   AND  ITS  MANAGEMENTS.pdf
PNEUMOTHORAX AND ITS MANAGEMENTS.pdfDolisha Warbi
 
Hematology and Immunology - Leukocytes Functions
Hematology and Immunology - Leukocytes FunctionsHematology and Immunology - Leukocytes Functions
Hematology and Immunology - Leukocytes FunctionsMedicoseAcademics
 
Presentació "Real-Life VR Integration for Mild Cognitive Impairment Rehabilit...
Presentació "Real-Life VR Integration for Mild Cognitive Impairment Rehabilit...Presentació "Real-Life VR Integration for Mild Cognitive Impairment Rehabilit...
Presentació "Real-Life VR Integration for Mild Cognitive Impairment Rehabilit...Badalona Serveis Assistencials
 

Último (20)

Presentation for Bella Mahl 2024-03-28-24-MW-Overview-Bella.pptx
Presentation for Bella Mahl 2024-03-28-24-MW-Overview-Bella.pptxPresentation for Bella Mahl 2024-03-28-24-MW-Overview-Bella.pptx
Presentation for Bella Mahl 2024-03-28-24-MW-Overview-Bella.pptx
 
Giftedness: Understanding Everyday Neurobiology for Self-Knowledge
Giftedness: Understanding Everyday Neurobiology for Self-KnowledgeGiftedness: Understanding Everyday Neurobiology for Self-Knowledge
Giftedness: Understanding Everyday Neurobiology for Self-Knowledge
 
Study on the Impact of FOCUS-PDCA Management Model on the Disinfection Qualit...
Study on the Impact of FOCUS-PDCA Management Model on the Disinfection Qualit...Study on the Impact of FOCUS-PDCA Management Model on the Disinfection Qualit...
Study on the Impact of FOCUS-PDCA Management Model on the Disinfection Qualit...
 
PERFECT BUT PAINFUL TKR -ROLE OF SYNOVECTOMY.pptx
PERFECT BUT PAINFUL TKR -ROLE OF SYNOVECTOMY.pptxPERFECT BUT PAINFUL TKR -ROLE OF SYNOVECTOMY.pptx
PERFECT BUT PAINFUL TKR -ROLE OF SYNOVECTOMY.pptx
 
Wessex Health Partners Wessex Integrated Care, Population Health, Research & ...
Wessex Health Partners Wessex Integrated Care, Population Health, Research & ...Wessex Health Partners Wessex Integrated Care, Population Health, Research & ...
Wessex Health Partners Wessex Integrated Care, Population Health, Research & ...
 
Nutrition of OCD for my Nutritional Neuroscience Class
Nutrition of OCD for my Nutritional Neuroscience ClassNutrition of OCD for my Nutritional Neuroscience Class
Nutrition of OCD for my Nutritional Neuroscience Class
 
MedDRA-A-Comprehensive-Guide-to-Standardized-Medical-Terminology.pdf
MedDRA-A-Comprehensive-Guide-to-Standardized-Medical-Terminology.pdfMedDRA-A-Comprehensive-Guide-to-Standardized-Medical-Terminology.pdf
MedDRA-A-Comprehensive-Guide-to-Standardized-Medical-Terminology.pdf
 
Case Report Peripartum Cardiomyopathy.pptx
Case Report Peripartum Cardiomyopathy.pptxCase Report Peripartum Cardiomyopathy.pptx
Case Report Peripartum Cardiomyopathy.pptx
 
CEHPALOSPORINS.pptx By Harshvardhan Dev Bhoomi Uttarakhand University
CEHPALOSPORINS.pptx By Harshvardhan Dev Bhoomi Uttarakhand UniversityCEHPALOSPORINS.pptx By Harshvardhan Dev Bhoomi Uttarakhand University
CEHPALOSPORINS.pptx By Harshvardhan Dev Bhoomi Uttarakhand University
 
LUNG TUMORS AND ITS CLASSIFICATIONS.pdf
LUNG TUMORS AND ITS  CLASSIFICATIONS.pdfLUNG TUMORS AND ITS  CLASSIFICATIONS.pdf
LUNG TUMORS AND ITS CLASSIFICATIONS.pdf
 
SGK HÓA SINH NĂNG LƯỢNG SINH HỌC 2006.pdf
SGK HÓA SINH NĂNG LƯỢNG SINH HỌC 2006.pdfSGK HÓA SINH NĂNG LƯỢNG SINH HỌC 2006.pdf
SGK HÓA SINH NĂNG LƯỢNG SINH HỌC 2006.pdf
 
Epilepsy
EpilepsyEpilepsy
Epilepsy
 
PULMONARY EDEMA AND ITS MANAGEMENT.pdf
PULMONARY EDEMA AND  ITS  MANAGEMENT.pdfPULMONARY EDEMA AND  ITS  MANAGEMENT.pdf
PULMONARY EDEMA AND ITS MANAGEMENT.pdf
 
Statistical modeling in pharmaceutical research and development.
Statistical modeling in pharmaceutical research and development.Statistical modeling in pharmaceutical research and development.
Statistical modeling in pharmaceutical research and development.
 
Big Data Analysis Suggests COVID Vaccination Increases Excess Mortality Of ...
Big Data Analysis Suggests COVID  Vaccination Increases Excess Mortality Of  ...Big Data Analysis Suggests COVID  Vaccination Increases Excess Mortality Of  ...
Big Data Analysis Suggests COVID Vaccination Increases Excess Mortality Of ...
 
COVID-19 (NOVEL CORONA VIRUS DISEASE PANDEMIC ).pptx
COVID-19  (NOVEL CORONA  VIRUS DISEASE PANDEMIC ).pptxCOVID-19  (NOVEL CORONA  VIRUS DISEASE PANDEMIC ).pptx
COVID-19 (NOVEL CORONA VIRUS DISEASE PANDEMIC ).pptx
 
Radiation Dosimetry Parameters and Isodose Curves.pptx
Radiation Dosimetry Parameters and Isodose Curves.pptxRadiation Dosimetry Parameters and Isodose Curves.pptx
Radiation Dosimetry Parameters and Isodose Curves.pptx
 
PNEUMOTHORAX AND ITS MANAGEMENTS.pdf
PNEUMOTHORAX   AND  ITS  MANAGEMENTS.pdfPNEUMOTHORAX   AND  ITS  MANAGEMENTS.pdf
PNEUMOTHORAX AND ITS MANAGEMENTS.pdf
 
Hematology and Immunology - Leukocytes Functions
Hematology and Immunology - Leukocytes FunctionsHematology and Immunology - Leukocytes Functions
Hematology and Immunology - Leukocytes Functions
 
Presentació "Real-Life VR Integration for Mild Cognitive Impairment Rehabilit...
Presentació "Real-Life VR Integration for Mild Cognitive Impairment Rehabilit...Presentació "Real-Life VR Integration for Mild Cognitive Impairment Rehabilit...
Presentació "Real-Life VR Integration for Mild Cognitive Impairment Rehabilit...
 

Pediatric endocrinology review questions

  • 1. Pediatric Endocrinology Review MCQs Abdulmoein Eid Al-Agha, FRCPCH Professor of Pediatric Endocrinology, Website: http://aagha.kau.edu.sa
  • 2. Five-year-old girl presented with bilateral breast enlargement & vaginal discharge, together with moodiness and body odor, no relevant past medical history. She was well with no headaches, visual disturbance or polydipsia. Mother and two elder sisters had early menarche at 10–11 years. On examination, her height is on the 90th% and mid-parental height 50th%. Tanner stage is of B3, PH2, A1. Which one of the following is most important investigation?
  • 3.
  • 4. a) Observation of further progression of pubertal signs. b) Bone age assessment. c) MRI pituitary to look for CNS tumor. d) Basal &GnRH stimulation test.
  • 5. Precocious Puberty • Central precocious puberty (CPP) is caused by an early activation of the hypothalamic-pituitary- gonadal axis. – CPP is pathologic in up to 40 - 75 % of boys & 10 - 20 % of girls. • Peripheral precocity is caused by: – secretion of sex hormones either from the gonads or adrenal glands, ectopic human chorionic gonadotropin (hCG) production by a germ-cell tumor, or by exogenous sources of sex steroids – Is independent from the hypothalamic-pituitary- gonadal axis
  • 7. Five- year old girl, brought by her mother because of bilateral breast enlargement and spotty vaginal discharges. On examination (see photo). Her basal pubertal investigations revealed: - Estradiol 62 pg/ml (<10) - FSH <0.1 mIU/mL - LH <0.1 mIU/mL
  • 8.
  • 9. Which one of the following, is the most important confirmatory investigation you will order? a) GnRH stimulation test. b) Gene mutation screening. c) Thyroid function test. d) Skeletal survey.
  • 10. McCune-Albright syndrome (MAS) consists of at least 2 of the following 3 features: • Polyostotic fibrous dysplasia (PFD). • Café-au-lait skin pigmentation, • Autonomous endocrine hyperfunction (e.g., gonadotropin-independent precocious puberty). • Other endocrinopathies may be present, including hyperthyroidism, acromegaly &Cushing syndrome. • Genetically, activating mutation of the GNAS1 gene , which is involved in G- protein signaling .
  • 13. A 2 year old girl with bilateral breast development with no growth acceleration, no bone age advancement and normal estradiol, LH or FSH. What is the most likely diagnosis ? a) Ingestion of her mother’s OCPs. b) Precocious puberty. c) Benign isolated premature thelarche. d) McCune Albright Syndrome.
  • 14. Benign Premature Thelarche • Isolated breast development – 80% before age 2 years. – Rarely after age 4 years. • Not associated with other signs of puberty. (growth acceleration, advancement of bone age) • Children go on to normal timing of puberty and normal fertility. • It may be associated with functional follicular cysts that spontaneously regress and perhaps with especially responsive breast tissue • Benign process. • Routine follow-up.
  • 15. Five- year old girl, with 6 months history of pubic hair growth associated with fine axillary hair as well as adult odor to sweat. No breast development with no acceleration of growth. Otherwise normal history and examinations. What is the most likely diagnosis? a) Precocious puberty. b) Benign premature Adrenarche. c) Non-classical congenital adrenal hyperplasia. d) Adrenal tumor.
  • 16. Benign premature adrenarche • Production of adrenal androgens before true pubertal development begins. • Presents as isolated pubic hair in mid childhood – No growth acceleration. – No testicular enlargement in boys. • If normal growth rate, routine follow-up. • If accelerated growth and/or bone age advancement, screen for: – CAH – Virilizing tumor (adrenal/gonadal)
  • 17. Four- year old girl, presented with abdominal distension, there is A three months history of bilateral breast enlargement, pubic hair appearances & rapid growth. Her abdominal ultrasound (photo). Which one of the following is most likely diagnosis? a) β-HCG secreting hepatoblastoma b) McCune Albright syndrome. c) Benign premature Adrenarche. d) Ovarian cyst.
  • 18. Gonadotropin independent precocious puberty secondary to β-HCG secreting hepatoblastoma
  • 19. Hepatoblastoma • Is the most common primary hepatic malignancy in early childhood. • The majority of cases occur in the first two years of life and rarely in children older than five years. • Syndromes with an increased incidence of hepatoblastoma include: – Beckwith Wiedemann syndrome. – trisomy 18 & trisomy 21 . – Aicardi syndrome. – Li-Fraumeni syndrome. – Goldenhar syndrome. – type 1a glycogen storage disease. – familial adenomatous polyposis. • Serum alpha-fetoprotein (AFP) levels are markedly elevated. • Sexual precocity may be present due to the synthesis of ectopic gonadotropin (HCG).
  • 20. Thirteen-year-old girl, is referred with growth failure with delayed puberty. On examination her height is below the 0.4th centile and her weight is on the 25th centile. (Please see photo). Which one of the following is most likely diagnosis? a) Constitutional delay of Puberty. b) Hypogonadotropic hypogonadism. c) Hypopituitarism. d) Hypergonadotropic hypogonadism.
  • 21.
  • 22.
  • 23. Ovarian failure • Turner syndrome is one of the most common causes of premature ovarian failure. • Most affected girls have no breast development and have primary amenorrhea. • Approximately 15 - 30 % of girls with Turner syndrome either have initial breast development followed by pubertal arrest, or complete puberty but then develop secondary amenorrhea. • A small percentage of girls have normal pubertal development and regular menstruation. • These milder phenotypes of ovarian failure are more common in girls with mosaicism compared with those with 45,X monosomy.
  • 24. A full term neonate, is born with isolated bilateral swollen feet. What is the most likely diagnosis? a) Cardiac failure with lower limb edema. b) Systemic allergic reaction. c) Congenital nephrotic syndrome. d) Turner syndrome.
  • 25. Turner syndrome • Combination of short stature, primary amenorrhea (ovarian dysgenesis), webbed neck, lymphedema, and cubitus valgus. • incidence among live born female infants of one in 5000. • More than half have a 45, X karyotype. • The remainder show mosaicism and/or more complex rearrangements involving the X chromosome. • Between 20% and 40% of girls with Turner syndrome have significant heart defects, most commonly coarctation of the aorta (70%), often bicuspid aortic valve, and aortic stenosis.
  • 27. Ten –year - old girl, presented with arm deformity (photo). What is the diagnosis?
  • 28. Madelung deformity • Madelung deformity is a focal dysplasia of the distal radial epiphysis. • Premature closure leads to a progressive deformity with dorsal displacement of the ulna. • Madelung deformity occurs in girls with Turner syndrome and is usually bilateral. • The diagnosis of Madelung deformity is made radiographically; radiographs of both wrists, forearms, and elbows should be obtained.
  • 30. Seven-year -old boy has presented to the endocrinology clinic with short stature & mental subnormality. What is the most likely diagnosis? a) Turner’s syndrome. b) VATER syndrome. c) Silver – Russel syndrome. d) Noonan’s syndrome.
  • 31. Noonan syndrome (NS) • Autosomal dominant disorder. • The classical features include: – short stature. – congenital heart disease (CHD). – clinical features of NS include difficulties with feeding in early life; vision, hearing, and growth problems; specific learning difficulties & easy bruising and bleeding.
  • 32. Ten- year female presented, with a 4 week history of polyuria, polydipsia, and unexplained weight loss. She was noticed to have deep, sighing respiration. Glucose was 498 mg/dL, pH 7.06. Which one of the following is the most important initial management ? a) Insulin drip 0.1 units/kg/hour. b) ½ Normal Saline with 40 meq K at 2x maintenance. c) Bicarbonate 1 meq/kg slowly over 1 hour. d) Fluid rehydration with 0.9 NS.
  • 33. Sixteen -year old boy, presented with delayed puberty. He was having a recurrent episodes of headache, diplopia and increased urination. His height was < 3rd percentile. Which one of the following is the most likely diagnosis? a) Diabetes mellitus. b) Cerebellar tumor. c) Craniopharyngioma. d) Kalman's syndrome.
  • 34. Craniopharyngioma • Benign tumor with tendency to recur after excision and the high surgical risk due to involvement of the most vital structures of the brain. • Rare solid or mixed solid & cystic tumors that arise from remnants of Rathke's pouch. • Historically, been named as "Rathke pouch tumors“. • A wide range of symptoms may be present. • Visual symptoms are frequent, result from compression of the optic chiasm or nerves. • Moderate to severe daily headaches are present in approximately 50 % of patients at the time of diagnosis. • Other generalized symptoms, such as depression, nausea, vomiting, and lethargy can accompany pressure-related headaches.
  • 35. Craniopharyngioma • Endocrine abnormalities due to direct damage to or compression of normal structures or post surgical removal of the tumor can lead to a range of endocrine abnormalities. • Frequently observed complications include diabetes insipidus (75%). • Growth failure, caused by either hypothyroidism or growth hormone deficiency, is the most common presentation in children. • Delayed puberty could be seen in 40 % of cases.
  • 37. Twenty – day- old, neonate was seen in the pediatric endocrinology clinic for recurrent episodes of hypoglycemia . On examination, he had a cleft lip and palate with a small mid-face. Both testes were palpable, however short penile length was measured. What is your best approach in order to reach to the final cause of hypoglycemia? a) Look for other dysmorphic features. b) Admit to do critical samples during his hypoglycemia attack. c) Do GH stimulation test. d) Do MRI brain.
  • 38.
  • 39.
  • 40. Holoprosencephaly (HPE) • Is cephalic disorder in which the forebrain of the embryo fails to develop into two hemispheres. • Normally, the forebrain is formed and the face begins to develop in the fifth and sixth weeks of gestation. • The condition can be mild or severe. • Most cases are not compatible with life and result in fetal death in-utero. • When the embryo's forebrain does not divide to form bilateral cerebral hemispheres (the left and right halves of the brain), it causes defects in the development of the face , brain & pituitary structure and function.
  • 41. Hypopituitarism • Affects between 1 in 4,000 -10,000 live births, with increasing incidence with age. • Congenital hypopituitarism most often results from genetic or embryologic pathologies. • The clinical presentation of hypopituitarism, widely varies, depends on the patient's age, the etiology, and the specific hormone deficiencies, which may occur as isolated deficiencies or in various combinations of MPHD.
  • 42. Hypopituitarism • Most neonates with hypopituitarism have normal birth weights and lengths and no history of intrauterine growth retardation. • However, they often have histories of breech presentation (particularly neonates with MPHD), although the explanation for this is unclear. • The hypoglycemia risk is higher in neonates with hypopituitarism, with various manifesting symptoms, such as lethargy, jitteriness, pallor, cyanosis, apnea, or convulsions. • Jaundice may be secondary to indirect hyperbilirubinemia (TSH deficiency) or to direct hyperbilirubinemia (GH or ACTH deficiencies).
  • 43. Ten-day old boy presented with hypertension & metabolic alkalosis. He was diagnosed with congenital adrenal hyperplasia. Which of the following enzyme deficiency could be the cause? a) Aldosterone synthase deficiency. b) 21-β- hydroxylase deficiency. c) 3-β-hydroxysteroid dehydrogenase deficiency. d) 17- hydroxylase deficiency.
  • 44.
  • 45. • 17-Hydroxylase (17-OH) deficiency syndrome is a rare genetic disorder of steroid biosynthesis . • It causes decreased production of glucocorticoids and sex steroids, resulting in 46,XY DSD. • Increased synthesis of mineralocorticoids precursors, resulting in varying degrees of hypertension and hypokalaemia • It is a rare form of congenital adrenal hyperplasia resulting from loss-of-function mutations involving the CYP17 gene. • Exogenous glucocorticoid therapy is the treatment of choice which suppresses adrenocorticotropic hormone (ACTH) secretion, decreases 11-DOC and corticosterone levels, and normalizes serum potassium and blood pressure.
  • 46. A neonate was born with abnormal newborn thyroid screen done at 3 days of age: fT4 was 8 pmol/l (12-22). TSH was 60 μIU/mL (0.3-5.0). Which one of the following might be the cause? a) Hypothyroidism due to dysgenesis of the thyroid gland. b) Central hypothyroidism. c) TBG deficiency. d) Hypothyroidism from excess iodine exposure.
  • 47. • Thyroid dysgenesis/agenesis is the commonest cause of congenital primary hypothyroidism in almost 90% of cases. • Prevalence 1 in 4,000. • Symptoms and signs include the following: – Decreased activity. – Large anterior fontanelle. – Poor feeding and weight gain. – Small stature or poor growth. – Jaundice. – Decreased stooling or constipation. – Hypotonia. – Hoarse cry.
  • 48.
  • 49.
  • 50. 6 month female with congenital hypothyroidism ..following 4 months therapy
  • 51. Thirteen -year-old - girl presented to the orthopedic clinic with two year history of a limping in her left leg. She has been diagnosed with hypothyroidism but not compliant on her medication. On examination she is short & obese with a goiter and other signs of hypothyroidism. She had limitation of movement of her left hip and a limp. What is the most likely diagnosis? a) Slipped capital femoral epiphysis (SCFE). b) Chronic Osteomyelitis. c) Vitamin D deficiency. d) Monoarticular rheumatoid arthritis.
  • 52. Slipped capital femoral epiphysis (SCFE) • Characterized by a displacement of the capital femoral epiphysis from the femoral neck through the physeal plate. • The mean age of presentation is 12 years in girls and 13.5 years in boys, near the time of peak linear growth. • Obesity is a significant risk factor. • The two most common features of the presentation are pain and altered gait. • The classic presentation is that of an obese adolescent with a complaint of chronic or intermittent non radiating, dull, aching pain in the hip, groin, thigh, or knee, and no history of preceding trauma. • The diagnosis is usually made on plain radiographs, which reveal an apparent posterior displacement of the femoral epiphysis, like ice cream slipping off a cone.
  • 53. Slipped capital femoral epiphysis (SCFE)
  • 54. Newborn with DSD, noted to have generalized hyperpigmentation. Pelvic ultrasound demonstrated the presence of a both ovaries &uterus. What is the most useful test for the diagnosis? a) Testosterone. b) 17-hydroxyprogesterone. c) Serum electrolytes. d) DHEAS.
  • 55. Congenital adrenal hyperplasia • It is autosomal recessive disorders. • Over 95 % of cases are due to 21-hydroxylase deficiency. • In many countries, neonatal screening for 21OHD is performed routinely in all newborns by measurement of 17OHP in a dried, filter paper blood spot. • The characteristic biochemical abnormality in patients with classic 21OHD is an elevated serum concentration of 17- hydroxyprogesterone (17OHP). • False-positive results are common with premature infants, and many screening programs have established reference ranges that are based upon weight and gestational age. • False-negative results may occur as a result of maternal antenatal glucocorticoid use, in first 2 days of life & in sick neonates.
  • 56. Cholesterol Pregnenolone Progesterone DOCA Corticosterone ALDOSTERONE 17 (OH) pregnenolone DHEA 17 (OH) progesterone Androstenedione Compound S CORTISOL TESTOSTERONE Desmolase 3--HSD 3--HSD 3--HSD 17-OH 17-OH 21-OH 21-OH 11-OH 11-OH
  • 57. Types of CAH • Classical type: simple virilizing. salt losing. • “Non classical” / Late onset.
  • 58. a) Assure & re-examination him again at age 18 months. b) Refer the patient for exploratory laparotomy. c) Measure basal & HCG stimulated testosterone & DHT. d) Start therapy with testosterone enanthate, 50 mg IM monthly for 3 months. Six-month old 46 XY infant presented with non palpable testes. Which one of the following, the most appropriate next step would be?
  • 59. Cryptorchidism • Usually is an isolated finding. • However, it may occur in association with endocrine disorders, genetic syndromes, &morphologic abnormalities, particularly if the cryptorchidism is bilateral. • Disorders of sex development (DSD) (e.g., mixed gonadal dysgenesis, Ovotesticular disorders of sex development, persistent Müllerian duct syndrome) • Bilaterally nonpalpable testes in a phenotypically male newborn require immediate evaluation at the time of birth because the differential diagnosis includes DSD.
  • 60. Fifteen year old, female has presented with primary amenorrhea. Her breast development began at 10 years of age. Current height was 160 cm, weight was 45 kg. Tanner stage of B3 PH 1-2. What is the most likely diagnosis? a) Mullerian dysgenesis. b) Turner’s syndrome. c) Complete androgen insensitivity syndrome. d) Kalman's syndrome.
  • 61. Delayed puberty is defined by: • In boys: – No enlargement of the testes (testicles) by age 14 years. – A time lapse of more than 5 years from the onset to the completion of growth of the genitals. • In girls: – No breast development by age 13 years. – A time lapse of more than 5 years from the beginning of breast growth to the first menstrual period. – No menstruation (amenorrhea) by age 16. • Delayed puberty could be either to hypogonadotropic hypogonadism or hypergonadotropic hypogonadism.
  • 62. • The most common cause of delayed puberty is hypogonadotropic hypogonadism “ functional defect” “ constitutional delay”. • Other functional defects, such as undernutrition, excessive exercise, anorexia or chronic illness. • Adolescents, particularly girls, who become very thin because of excessive exercise or dieting often have delayed puberty, including an absence of menstruation. • Kallmann syndrome is one of genetic disorders, with isolated gonadotropin deficiency (without affecting production of other hormones). • Hypergonadotropic hypogonadism in boys, could be due to twisting of a testis (testicular torsion), or infection (such as mumps) or, chromosomal abnormalities, such as Turner syndrome in girls and Klinefelter syndrome in boys.
  • 63. Six-year-old girl is referred with growth failure, poor appetite, recurrent abdominal pain, ‘thick custard’ stools and vomiting. What is the appropriate most confirmatory investigation? a) Bone age. b) Anti-tissue transglutaminase antibody. c) Jejunal biopsy. d) Serum iron & ferritin.
  • 64. Jejunal biopsy is confirmatory investigation of celiac disease, which will reveal villous atrophy & lymphocytic infiltrate in the lamina propria with hyperplastic crypts.
  • 65. If a child presented with abdominal distension & short stature, think of celiac disease !
  • 66. Celiac disease • Celiac disease is a small bowel disorder characterized by mucosal inflammation, villous atrophy, and crypt hyperplasia, which occur upon exposure to dietary gluten and demonstrates improvement after withdrawal of gluten from the diet. • Children with celiac disease may exhibit no unusual findings on physical examination. • Sometimes, protuberant abdomen & marked muscle wasting and evidence of malnutrition. • Usually, they responded very well to a strict gluten‐free diet that she must follow for the rest of her life.
  • 67. Celiac disease • Evaluation begins with serologic testing. • Patients with positive serologic testing, should undergo an upper endoscopy with small bowel biopsy to diagnose celiac disease. • IgA - anti-tissue transglutaminase (tTG) antibody is the single preferred test for detection of celiac disease. • In addition, total IgA levels should be measured. • In patients with IgA deficiency, IgG-deamidated gliadin peptide (DGP) IgG.
  • 68. Eight –year - old girl, has presented with goiter, abnormal thyroid function test of high TSH, low fT4. She has sensorineural hearing loss. What is the most likely inheritance of her disease? a) Autosomal recessive. b) Autosomal dominant. c) Sex-linked dominant. d) Sex –linked recessive.
  • 69. Pendred syndrome • It is inherited in an autosomal recessive. • Is caused by a mutation in the SLC26A4 gene, which causes a defect in the organification of iodine (i.e., incorporation into thyroid hormone), congenital sensorineural hearing loss, and, usually, an enlarged thyroid gland. • combination of congenital bilateral sensorineural hearing loss & goiter with euthyroid or mild hypothyroidism. • There is no specific treatment, other than supportive measures for the hearing loss and thyroxine in case of hypothyroidism.
  • 70. Twelve-year old girl, who has presented with exophthalmos and weight loss. Her laboratory tests confirm Graves disease. Which most likely cause for her exophthalmos? a) Fat tissue deposition. b) Ocular muscle hyperplasia. c) Inflammatory infiltrate. d) Globe enlargement.
  • 71. Graves Ophthalmopathy • A gritty or foreign object sensation in the eyes. • Excessive tearing that is often made worse by exposure to cold air, wind, or bright lights. • Eye or retroocular discomfort or pain. • Blurring of vision. • Diplopia. • Occasionally decreased vision.
  • 72. Graves' orbitopathy (ophthalmopathy) • Is an autoimmune disease of the retroorbital tissues. • There is no much evidence that RAI causes or worsens Graves ophthalmopathy in children or adolescents, as compared to adults. • children with active, mild orbitopathy may still be candidates for anti-thyroid, radioiodine, or surgery. • children with active, moderate-to-severe orbitopathy, anti-thyroid or surgery are the preferred treatment options. • Patients who refuse surgery and who have contraindications to anti-thyroid may need to be offered radioiodine therapy with steroid coverage.
  • 73. Graves' orbitopathy (ophthalmopathy) • Prednisone seems a better choice than intravenous corticosteroids, for treating moderate-to-severe Graves’ ophthalmopathy. • Prednisone (1 mg/kg/day) for four weeks, followed by gradual tapering over another 2-3 months depending on severity and response. • For patients treated surgically, total thyroidectomy is the procedure of choice. • Total thyroidectomy eliminates more thyroid antigens than subtotal thyroidectomy, resulting in progressive decreases in antibodies to all major thyroid antigens and especially to the thyrotropin receptor. • Total thyroidectomy was associated with less progression of proptosis postoperatively than subtotal thyroidectomy.
  • 74. Disrupted muscle fibers and an area of inflammatory cell accumulation in the superior rectus muscle of a patient with ophthalmic Graves' disease. The extra ocular muscles are swollen, and some muscle fibers show loss of striation, fragmentation, and infiltration with lymphocytes, most of which are T lymphocytes.
  • 75.
  • 76. Ten – year old girl with graves disease, on carbimazole therapy. Which one of the following is the most serious side effect of anti -thyroid medication? a) Fever. b) Rash. c) Granulocytopenia. d) Arthritis.
  • 77. Anti-Thyroid medications • Used for long-term control of hyperthyroidism in children, adolescents, and pregnant women • Methimazole is more potent and longer-acting than propylthiouracil. • Propylthiouracil is reserved for use in thyroid storm, first trimester of pregnancy, and methimazole allergy or intolerance. • Patients with Graves disease may experience remission after treatment for 12-18 months, but recurrences are common within the following year. • Toxic multinodular goiter & toxic adenoma will not go into remission.
  • 78. Anti-thyroid therapy • Thionamide compounds were found in 1943 to inhibit thyroid hormone synthesis. • Methimazole (MMI) is first choice of anti- thyroid therapy. • PTU is associated with risk of severe hepatotoxicity. • Anti –thyroid medications, inhibit both organification of iodine to tyrosine residues in thyroglobulin & coupling of iodothyronines. • Adverse effects of anti- thyroid therapy include, minor side effects (e.g., rash, fever, arthritis) and rare serious adverse effects, such as agranulocytosis & hepatotoxicity.
  • 79. • If remission does not occur following ATD therapy, 131I or surgery should be contemplated. • When 131I is administered, dosages should be greater than 150 uCi/gm of thyroid tissue, with higher dosages needed for large glands. • When surgery is performed near total or total- thyroidectomy is the recommended procedure. • Complications for thyroidectomy in children are considerably higher than in adults, thus an experienced thyroid surgeon is needed when children are operated on.
  • 80. Eleven- year- old boy presented with severe proptosis , tachycardia & fine tremor. He is confirmed to have Graves disease. Which of the following is most accurate statement? a) Radioactive iodine therapy in this case is first line of management. b) PTU is more preferable than MTZ. c) Granulocytopenia is a complication of anti thyroid medications. d) Surgery is a good option for this child.
  • 81. Sixteen- year - old boy presented with thyrotoxicosis with bilateral, firm, non- pitting, asymmetrical plaques over his legs (see photo).
  • 82. Which one of the following describes skin lesion? a) Pretibial myxedema. b) Scleroderma. c) Eczematous lesions. d) Dermatomyositis.
  • 83. Pretibial myxedema (PTM) • PTM or thyroid dermopathy is a term used to describe localized lesions of the skin resulting from the deposition of hyaluronic acid, usually as a component of thyroid disease. • Thyroid dermopathy occurs rarely. • Although PTM is most often confined to the pretibial area, it may occur anywhere on the skin, especially the ankle, dorsum of the foot, knees, shoulders, elbows, upper back, pinnae, nose, and neck. • It is nearly always associated with autoimmune thyroid disease.
  • 84. Six- year - old boy with obesity, mental retardation and subcapsular cataract. Which one of the following is expected laboratory finding? a) Low calcium, low phosphate b) High calcium, high phosphate c) Low calcium, high phosphate d) Normal calcium, normal phosphate
  • 85.
  • 86. Pseudohypoparathyroidism (PHP) • In 1942, Fuller Albright first introduced the term Pseudohypoparathyrodism to describe patients who presented with PTH-resistant hypocalcaemia and hyperphosphatemia along with skeletal defects, collectively termed “Albright hereditary osteodystrophy (AHO)”. • These features included short stature, rounded face, shortened fourth metacarpals of hands and feet, obesity, dental hypoplasia, and soft-tissue calcifications. • Results from end organ resistance(renal & bone) to parathyroid hormone (PTH). • Characterized by hypocalcemia, hyperphosphatemia, & elevated PTH level.
  • 87. Pseudohypoparathyroidism (PHP) • There are several subtypes of PHP, based upon genetic and clinical characteristics including. – type 1a – type 1b – type 1c (PHP-1c – type 2 (PHP-2) – pseudo pseudohypoparathyroidism (PPHP). • PHP type 1a is the most common subtype and represents 70% of cases. • AHO phenotype is not a feature of PHP-1b or PHP-2.
  • 88. Thirteen- year old girl with obesity, retinitis pigmentosa and hypogonadism. Which of the following is most likely diagnosis? a) Prader –Willi syndrome. b) Laurence- Moon- Biedl syndrome. c) Lowe oculo-cerebral syndrome. d) Familial cerebellar ataxia.
  • 89. Genetic syndromes associated with childhood obesity include the following: • Prader-Willi syndrome. • Pseudohypoparathyroidism. • Laurence-Moon-Biedl (Bardet-Biedl) syndrome • Cohen syndrome. • Down syndrome. • Turner syndrome.
  • 90. Bardet–Biedl syndrome (BBS) • A rare autosomal recessive syndrome • Characterized by retinitis pigmentosa, obesity, post-axial polydactyly, renal dysfunction, learning difficulties and hypogonadism. • The diagnosis is based on clinical findings and can be confirmed by sequencing of known disease-causing genes in 80% of patients.
  • 92. Eleven- year - old boy, presented with truncal obesity, striae, hypertension & osteoporosis. Which one of the following is the commonest cause? a) Adrenal adenoma. b) Adrenal carcinoma. c) Bilateral adrenal hyperplasia d) Lipoid adrenal hyperplasia
  • 94.
  • 95. ACTH-independent Cushing syndrome • Overproduction of glucocorticoid may be due to an adrenal adenoma, adrenal carcinoma, or macronodular or micronodular adrenal hyperplasia. • The zona fasciculate and zona reticularis layers of the adrenal cortex normally produce glucocorticoid and androgens. • Glucocorticoid-secreting tumors are derived from these cells and, thus, may secrete both glucocorticoid and androgens. • In general, excess androgen secretion is suggestive of an adrenal carcinoma rather than an adrenal adenoma. • These glucocorticoid-producing tumors do not usually secrete Aldosterone, which is produced in the zona glomerulosa layer of the adrenal cortex.
  • 96. ACTH-independent Cushing syndrome • Cushing syndrome may be overt, subclinical, cyclical, or periodic. • Primary bilateral macronodular adrenal hyperplasia is uncommon and characterized by multiple nonpigmented nodules that are greater than 10 mm in diameter and enlarged adrenal glands. • The exact etiology of this condition is not quite clear, however, genetic mutations, paracrine ACTH secretion, and aberrant hormone receptors have been reported to play a role in its pathogenesis. • McCune-Albright syndrome is a rare cause of precocious puberty. – It is associated with hyperfunction of the adrenal glands that may lead to Cushing syndrome. • Ectopic cortisol secretion from a case of ovarian carcinoma has been reported as a cause of ACTH independent Cushing syndrome.
  • 97. Ten-year old boy with Cushing syndrome presented acutely in the emergency room with Addisonian crisis. Which statement is most accurate? a) This is impossible, as diagnosis must be inaccurate. b) The patient should be reinvestigated again. c) The patient should be treated as Addisonian crisis. d) The patient is likely to have mixed pituitary tumor causing both diseases.
  • 98. • The most common cause of Cushing syndrome in children is prolonged exposure to steroids. • This often lead to suppression of H-P-A axis, so patient will not be able to produce cortisol in stress situations. • This result in Addisonian crisis if steroid doses were not increased during stress.
  • 99. Eleven - year old girl, has presented with headache, associated with hypertension. Her electrolytes revealed hypokalemia and alkalosis. Which one of the following is most likely diagnosis? a) Diuretic therapy. b) Bartter syndrome. c) Gitelman syndromes. d) Conn’s syndrome.
  • 100. Conn’s syndrome • The classic presenting signs of Conn's syndrome are hypertension & hypokalemia. • The most common subtypes of primary Aldosteronism are: – Aldosterone-producing adenomas (APAs) – Bilateral idiopathic Hyperaldosteronism (IHA; bilateral adrenal hyperplasia) – Less common forms include familial Hyperaldosteronism (FH) types I to IV. • Unilateral adrenal hyperplasia. • Aldosterone-producing adrenocortical carcinoma. • Ectopic aldosterone-producing tumor.
  • 101. Sixteen -year old boy presented with Gynecomastia. Which of the following medications could be the cause? a) Hydralazine. b) Tamoxifen. c) Spironolactone. d) Danazol.
  • 103. Fifteen – year old boy, has presented with tender gynecomastia (3 cm in diameter bilaterally). He is in early to mid puberty. Which one of the following is first option in the management? a)Treatment with an anti-estrogen (Tamoxifen). b)Treatment with an aromatase inhibitor. c)Surgery. d)Reassurance.
  • 104. • Gynecomastia is the benign proliferation of glandular breast tissue in males. • Pubertal Gynecomastia is common from puberty till age of 25 years. • It resolves spontaneously without any intervention. • It is most likely due to aromatization of testosterone into estrogen.
  • 105. A thorough history should be obtained that addresses the following: • Age of onset and duration of the condition • Any recent changes in nipple size and any pain or discharge from the nipples • History of mumps, testicular trauma, alcohol use, or drug use • Family history of Gynecomastia • History of sexual dysfunction, infertility, or hypogonadism
  • 106. Physical examination should include the following: • Thorough examination of the breasts, with attention to size and consistency • Assessment for any nipple discharge or axillary lymphadenopathy • Testing to differentiate between true Gynecomastia and pseudogynecomastia • Assessment of glandular tissue • Examination of the testicles, with attention to size and consistency, as well as nodules or asymmetry • Observation of any signs of feminization • Checking for any stigmata of chronic liver disease, thyroid disease, or renal disease
  • 107. Approach Considerations • Generally, no treatment is required for physiologic Gynecomastia. • Pubertal Gynecomastia resolves spontaneously within several weeks to 3 years in approximately 90% of patients. • Breasts greater than 4 cm in diameter may not completely regress. • If hypogonadism (primary or secondary) is the cause of Gynecomastia, parenteral or transdermal testosterone replacement therapy is instituted. • However, testosterone does have the potential to exacerbate Gynecomastia through the aromatization of the exogenous hormone into estradiol. • idiopathic Gynecomastia or with residual Gynecomastia after treatment of the primary cause, medical or surgical treatment may be considered.
  • 108. Pharmacologic Therapy • With the administration of clomiphene, an antiestrogen, approximately 50% of patients achieve partial reduction in breast size, and approximately 20% of patients note complete resolution. – Adverse effects, while rare, include visual problems, rash, and nausea. • Tamoxifen, an estrogen antagonist, is effective for recent- onset and tender Gynecomastia. • Up to 80% of patients report partial to complete resolution. Nausea and epigastric discomfort are the main adverse effects. • Other drugs used, albeit less frequently, include danazol. Danazol, a synthetic derivative of testosterone, inhibits pituitary secretion of LH and follicle-stimulating hormone (FSH), which decreases estrogen synthesis from the testicles.
  • 109. Sixteen –year- old adolescent, noted to have firm mass behind the right nipple. The mass is mobile, non tender. Select the most likely diagnosis? a) Lymphoma. b) Intraductal papilloma. c) Gynecomastia. d) Benign cyst.
  • 110. • Gynecomastia is a benign enlargement of the male breast (usually bilateral but sometimes unilateral) resulting from a proliferation of the glandular component of the breast. • It is defined clinically by the presence of rubbery or firm mass extending concentrically from the nipples. • Gynecomastia should be differentiated from pseudo gynecomastia (lipomata), which is characterized by fat deposition without glandular proliferation.
  • 111. Seven - years old boy has presented with vomiting, constipation, lethargy & excessive urination. No dysmorphic signs were noted. His serum calcium was 2.9 mmol/l (2.1-2.5), phosphate 0.8 (1.1-1.8), urinary calcium: creatinine ratio was high, PTH level was100 (6- 15) &vitamin D was 77 (75-125). Which one of the following is the most likely cause? a) William’s syndrome. b) Familial hypocalciuric hypercalcemia. c) Hypervitaminosis D. d) Parathyroid adenoma.
  • 112. Hypercalcemia in children • It may present with hypotonia, poor feeding, vomiting, constipation, abdominal pain, lethargy, polyuria, dehydration, failure to thrive and seizures. • In severe cases renal failure, pancreatitis and reduced consciousness may also occur. • In adolescents may present with psychiatric symptoms. • The causes of hypercalcemia in children can be classified as PTH‐dependent or PTH‐independent. • Could be congenital or acquired.
  • 113. Hypercalcemia in children • PTH‐independent hypercalcemia: – hypervitaminosis D. – granulomatous disorders. – idiopathic infantile hypercalcemia. – William's syndrome. • PTH‐dependent hypercalcemia: – parathyroid tumors, which may give rise to primary hyperparathyroidism (PHPT). – tertiary hyperparathyroidism, which usually arises in association with chronic renal failure. – treatment of hypophosphatemic rickets.
  • 114. • In approximately 85% of cases, primary hyperparathyroidism is caused by a single adenoma. • In 15% of cases, multiple glands are involved (i.e., either multiple adenomas or hyperplasia). • Rarely, primary hyperparathyroidism is caused by parathyroid carcinoma. • Familial cases can occur as either part of the multiple endocrine neoplasia syndromes (MEN 1 or MEN 2a), hyper parathyroid-jaw tumor (HPT-JT) syndrome, or familial isolated hyperparathyroidism (FIHPT). • Familial hypocalciuric hypercalcemia and neonatal severe hyperparathyroidism also belong to this category.
  • 115. Otherwise healthy 6-week infant presented with generalized seizure. She was, somewhat sleepy with normal examinations. Her glucose was 4.1 mmol/l, sodium 141 mmol/l, calcium 1.5 mmol/l, phosphorus 2.1 mmol/l, magnesium 0.8 mmol/l & ALP 200 IU/l. Which one of the following is most likely diagnosis? a) Pseudohypoparathyroidism. b) Hypoparathyroidism. c) Vitamin D deficiency. d) Albright’s hereditary osteodystrophy.
  • 116. • Hypoparathyroidism may be transient, inherited, or acquired. • Transient hypoparathyroidism: – Preterm and low birth neonates are at increased risk, and as many as 50% of them might have a deficient surge in PTH that results in hypocalcemia. – It is possible that the functional immaturity is a manifestation of a delay in development of the enzymes that convert glandular PTH to secreted PTH.
  • 117. • Permanent hypoparathyroidism: – DiGeorge syndrome (hypoparathyroidism, absence of thymus gland with T-cell abnormalities, and cardiac anomalies) is associated with abnormal development of the third and fourth pharyngeal pouches from which the parathyroid derive embryologically and represents an example of a defect in parathyroid gland development. – DiGeorge syndrome and velocardiofacial syndrome are variants of the chromosome arm 22q11 micro deletion syndrome.
  • 118. • X-linked recessive hypoparathyroidism has been associated with parathyroid agenesis and has been mapped to chromosome arm Xq26-q27, the location of a putative developmental gene. • Familial cases of hypoparathyroidism due to mutations of the PTH gene located on chromosome arm 11p15 have been identified. • These mutations have been both dominantly and recessively inherited.
  • 119. Eleven year old boy, presented with mental sub - normality, facial dysmorphism (photo), and severe growth failure. Which diagnostic laboratory investigation, you are going to order? a) IGF- 1. b) GH provocative test. c) Bone age. d) Serum calcium & PTH.
  • 120. • Sanjad Sakati syndrome (SSS) is an autosomal recessive disorder found exclusively in people of Arabian origin. • It is first reported from the Kingdom of Saudi Arabia in 1988 as a newly described syndrome mainly from the Middle East and the Arabian Gulf countries. • Children affected with this condition are born small for gestational age and present with hypocalcemic tetany or seizures due to hypoparathyroidism at an early stage in their lives. They have typical physical features, namely; long narrow face, deep set small eyes, beaked nose, large floppy ears, micrognathia, severe failure to grow both intrauterine and extra uterine and mild to moderate mental retardation.
  • 121. seventeen - year old adolescent, presented with hirsutism, irregular menses and acne. Which of the following supports the diagnosis of PCOS? a) High FSH level. b) LH: FSH ratio of 3:1. c) LH: FSH ratio of 1:1. d) High estradiol level.
  • 122. Definitions • Primary amenorrhea: Lack of menses by 15 years of age or three years after the onset of breast development. • Secondary amenorrhea: More than 90 days without a menstrual period, after previously menstruating. • Irregular menstruation cycle: defined as if it is less than 24 days or more than 38 days. • Irregular menstruation is common in adolescence. • A regular menstrual cycle can be set within a year of first menses.
  • 123. PCOS • Most expert groups use Rotterdam criteria to make the diagnosis of PCOS. • Two out of three of the following criteria are required to make the diagnosis: – Oligo- and/or anovulation. – Clinical and/or biochemical signs of hyperandrogenism. – Polycystic ovaries (by ultrasound). • The diagnosis of PCOS is confirmed once other conditions with features similar to PCOS have been excluded, such as: – non-classic congenital adrenal hyperplasia, hypothyroidism, hyperprolactinemia, androgen-secreting ovarian and adrenal tumors.
  • 124. • Polycystic ovary syndrome (PCOS), frequently becomes manifest during adolescence, and is primarily characterized by ovulatory dysfunction and hyperandrogenism. • The syndrome is heterogeneous clinically and biochemically. • The diagnosis of PCOS has lifelong implications with increased risk for metabolic syndrome, type 2 diabetes mellitus, and possibly cardiovascular disease and endometrial carcinoma. • PCOS should be considered in any adolescent girl presenting with a chief complaint of hirsutism, treatment-resistant acne, menstrual irregularity, acanthosis nigricans, and/or obesity
  • 125. Fifteen –year – old, obese adolescent female who has hirsutism & acne. Which one of the following, is most likely diagnosis? a) Cushing syndrome. b) Polycystic ovarian syndrome. c) Virilizing adrenal tumor. d) Non-classical CAH.
  • 126. Hirsutism versus virilization • Virilization includes clitoromegaly, male‐pattern baldness, deepening of the voice, and increased muscle mass in addition to the clinical features of hirsutism and chronic anovulation • The magnitude or amount of excessive hair growth can be approximated by the Ferriman‐Gallwey score • With this method, the amount of hair growth in nine androgen‐dependent areas is compared to a standard chart (grades 1 to 4) from which a score is derived. • Grade 1 indicates minimal terminal hair growth and grade 4 indicates dense terminal hair growth. • Scores greater than 8 are considered to indicate hirsutism
  • 128. Hypertrichosis / Hirsutism • Generalized hypertrichosis is a common adverse side effect of several medications. • Starvation, whether due to malnutrition or anorexia nervosa, and hypothyroidism can cause acquired hypertrichosis. • Several genetic disorders can be associated with excessive generalized hair growth. • Leprechaunism is characterized by hypertrichosis and insulin resistance due to mutations in the insulin receptor gene • Hirsutism is defined as excessive growth of coarse terminal hairs in androgen‐dependent areas such as the face, upper chest, abdomen, and back • Hypertrichosis is excessive hair growth in both androgen‐dependent & androgen‐independent areas.
  • 129. Thirteen year old adolescent presented with hirsutism. Which one of the following medications is therapeutic? a) Thiazide diuretic. b) Oral contraceptives. c) Sildenafil. d) Minoxidil.
  • 130. • Contraceptives can treat hirsutism in 2/3 of cases. • Direct suppression of ovarian steroid production and increase hepatic binding globulin production, which binds circulating androgens which lead to decrease free androgens.
  • 131. Eighteen months old boy was referred for further assessment of his increasingly bowing of legs. His parents are first degree cousins. He has been on vitamin D3 therapy for last 6 months, dose of 3000 unit/day with good compliance. His serum calcium was 1.37mmol/L, phosphate 0.13mmol/L , alkaline phosphatase 805IU/L, PTH 100 (6-15) pmol/L. Which one of the following is a diagnostic investigation? a) Do wrist x- ray to confirm active rickets. b) Do 25- hydroxy vitamin D metabolite. c) Do both 25 &1,25- di hydroxy vitamin d metabolites. d) Repeat bone profile next day after correction of hypocalcemia.
  • 133. HRVD type 2 • It is a very rare form of rickets. • Reported in approximately 100 cases only so far worldwide. • HRVD is an autosomal recessive disorder. • It is associated with end–organ resistance to 1,25- dihydroxyvitamin D (1,25[OH]2D). • Usually caused by mutations in the gene encoding the vitamin D receptor. • The defect in the receptor interferes with the function of the hormone-receptor complex, thereby preventing 1,25(OH)2D action.
  • 134. HRVD type 2 • Affected children usually appear normal at birth. • Develop rickets within the first two years of life. • Alopecia and ectodermal anomalies resulting from the lack of vitamin D receptor activity within keratinocytes develops in approximately two- thirds of cases and is a marker of disease severity (HRVD type 2A). • Other patients without alopecia or other ectodermal anomalies (HRVD type 2B).
  • 136. Twelve –year old boy, who has presented with height below 3rd.%, with growth velocity of 3 cm/ year. Both parents were of average height (MPH was 168 cm between 10th-25th%). On examination, Tanner staging was B3 PH 2. Among which one of the following is most appropriate approach? a) Observe & follow up in next six months. b) Re-assure the family, shortly she will catch- up. c) Do bone age only. d) Do full hormonal assessments workup.
  • 137. Short stature • Is defined as height that is 2 standard deviations (SD) or more below the mean height for children of that sex and chronologic age in a given population. • The clinical significance of short stature depends on many factors, including genetic potential and height velocity . • Determination of the child's growth (HV) is essential component of the evaluation for short stature. – Age two to four years – HV less than 5.5 cm/year – Age four to six years – HV less than 5 cm/year – Age six years to puberty: HV less than 4 cm/year for boys & less than 4.5 cm/year for girls.
  • 138. • Target height female = ((Height of father - 13) + Height of mother) / 2 ±8.5 cm. • Target height male = ((Height of mother + 13) + Height of father) / 2 ±8.5 cm. • Family history of growth & pubertal onset. • Review of systems for features suggestive of gastrointestinal, pulmonary, immunologic, or other systemic disease. • Dysmorphic features, especially webbed neck, cubitus valgus, and absent puberty in girls (suggests Turner syndrome), or disproportionate short stature (i.e., short limbs compared with trunk) suggestive of skeletal dysplasia.
  • 139. • Laboratory evaluation for a child with short stature depends on clinical evaluation. • The bone age used to the estimate for the child's adult height and in the evaluation for possible causes of short stature. • Children with pathological short stature should be evaluated with CBC, ESR, tTG IgA, creatinine, electrolytes, TSH, free thyroxine, IGF-1, and IGFBP-3. • A karyotype to evaluate for Turner syndrome. • Skeletal survey for children suspected skeletal dysplasia. • Clonidine, arginine and glucagon are common choices in children.
  • 140. • Hypothyroidism should be excluded first by performing thyroid function tests. • GH stimulation testing is not necessary for selected patients in whom other clinical criteria are sufficient to make the diagnosis of GHD, including: – Those with a known pituitary abnormality (congenital anomaly, tumor, or irradiation). – Those with known deficiency of at least one other pituitary hormone, with marked growth failure. • If GHD is confirmed, MRI of the hypothalamic-pituitary area is recommended to: – rule out tumors. – investigate for structural causes of GHD. – to evaluate the severity & prognosis of the deficiency.
  • 141. Six- year old boy, known case of growth hormone deficiency referred from orthopedic clinic for second opinion, whether or not to continue or not, as he has thoraco-lumbar scoliosis. Which one of the following is the best opinion? a) No worries, and continue GH safely. b) Stop GH therapy immediately. c) Decrease the dose and continue GH. d) Continue with close observation, hand with hand with orthopedic surgeon.
  • 142. Scoliosis & GH therapy • Scoliosis related to rapid growth that occurs with therapy and is not a direct effect of the growth hormone. • Patients with scoliosis who are treated with growth hormone should have their scoliosis monitored during therapy. • No clear statement to stop GH therapy, rather close monitoring during therapy.
  • 143. Contraindications of GH therapy • GH should not be used for pediatric treatment if patient's growth plates (epiphyses) are closed. • Active proliferative or severe non-proliferative diabetic retinopathy. • GH should not be used in patients with any evidence of any tumor. • GH should not be used in patients pre-existing or active malignancy.
  • 144. Contraindications of GH therapy • GH should not be used in patients with complication due to open heart or abdominal surgery and with acute respiratory failure. • GH is contraindicated in severely obese patients or with respiratory impairments. • GH is contraindicated in patients with Prader- Willi syndrome who are severely obese or have severe respiratory impairment.
  • 145. Which one of the following is not proven adverse effect of GH replacement therapy? a) Carpal tunnel syndrome. b) Arthralgia and myalgia. c) Benign intracranial hypertension. d) Increase incidence of brain tumor.
  • 146. Increase incidence of brain tumor is never documented & is not proven adverse effect of growth hormone therapy.
  • 147. GH side effects • Allergic reaction. • Ongoing injection site discomfort. • Pain in wrist (carpal tunnel). • Curvature of the spine (scoliosis). • Joint pain. • Puffy hands and/or feet (caused by fluid retention). • Changes in vision, headache, nausea. • Hip or knee pains. • Limping.
  • 148. Eight-year-old girl was referred because of tall stature. She has on & off headache. On examination there were no dysmorphic features. Her height is just above the 99th % and her parents’ MPH between 50th to 75th %. Tanner staging was: B2, PH 3, no menarche. Bone age was 12.4 years. Her final height prediction is 188 cm. Which one of the following statement is most appropriate?
  • 149. a) Most likely familial need to observe growth velocity. b) Need to do basal & stimulated GH test. c) Need to do IGF -1. d) Need to do oral glucose tolerance test for GH suppression.
  • 150. Tall stature • Children and adolescents with a height in excess of two standard deviations above the mean height for age are considered tall. • Obesity can also lead to tall stature in childhood. However, in such children, puberty usually occurs early, with a resulting normal final adult height. • Other causes of tall stature can include excessive secretion of growth hormone or sex hormones • In the presence of excessive sex hormones, there will be early growth acceleration, initially leading to tall stature, subsequently, there will usually be early epiphyseal closure, resulting in short stature after puberty.
  • 151. • An increased height velocity and advanced bone age suggest excessive production of sex hormones, growth hormone, or, occasionally, thyroid hormone. • Familial tall stature is a normal variant and is associated with normal height velocity and tall parents • Long extremities are associated with homocystinuria, Marfan’s syndrome, and Y chromosome disorders, whereas normal extremities are found in estrogen insensitivity and the other syndromes associated with tall stature. • More girls than boys seek medical attention for tallness because of the perceived negative social consequences of the condition. • In the rare instances in which treatment is indicated, relatively high doses of sex steroids are introduced early to advance osseous maturation.
  • 152. Six month old male infant presented with failure to thrive, constipation. His mother is complaining of too many diaper change and urine is leaking out of diapers most of the time. On examination he is having moderate to severe dehydration. His initial sodium is high 175 mmol/l., very low urine osmolality. Which one of the following is least common cause in the differential diagnosis of this infant?
  • 153. a) Langerhans cell histiocytosis. b) X-linked recessive nephrogenic DI. c) DIDMOAD syndrome. d) Psychological polydipsia.
  • 154. Central DI • The three most common causes of cranial diabetes insipidus are: – brain tumor that damages the hypothalamus or pituitary gland, which accounts for 25% of cases – severe head injury that damages the hypothalamus or pituitary gland, which accounts for 16% of cases – complications that occur during brain surgery, which account for 20% of cases
  • 155. Central DI • Less common causes of cranial diabetic insipidus include: – cancers that spread from another part of the body, such as the lungs or the bone marrow, to the brain – Wolfram syndrome, a rare genetic disorder that also causes sight and vision loss – brain damage caused by a sudden loss of oxygen, which can occur during a stroke or drowning – infections, such as meningitis and encephalitis, that can damage the brain
  • 156. Which one of the following is first line treatment of acute hypercalcemia? a) Calcitonin. b) Diuretics. c) Intravenous hydration. d) Bisphosphonate therapy.
  • 157. Which one of the following is commonest cause of 46 XX DSD? a) Partial AIS. b) CAH (21- OH deficiency). c) Virilizing ovarian or adrenal tumors. d) Placental aromatase enzyme deficiency.
  • 159. Which one of the following is commonest cause of 46 XY DSD? a) Testicular Hypoplasia. b) Partial AIS. c) Testosterone biosynthesis defects. d) 5  - Reductase deficiency.
  • 160. Three years old boy presented with goiter, short stature, deafness and symptoms suggestive of mild hypothyroidism. On examination, is having normal mentality, diffuse goiter, deaf and mute with normal CNS examination apart from sluggish reflexes. His bone age is retarded and has raised level of circulating TSH, fT4 and fT3. Which of the following is most likely diagnosis? a) Generalized resistance to thyroid hormone (GRTH) b) Pituitary resistance to thyroid hormone (PRTH) c) Pendred's syndrome d) TSH secreting Adenoma
  • 161. Resistance to thyroid hormone (RTH) • Is usually dominantly inherited • Characterized by elevated fT3 &fT4 and failure to suppress TSH secretion. • Variable refractoriness to hormone action in peripheral tissues. • Two major forms: – asymptomatic individuals with generalized resistance (GRTH). – patients with thyrotoxicosis features, suggesting predominant pituitary resistance (PRTH).
  • 162. • Recognized features of RTH include failure to thrive, growth retardation , ADHD in childhood, goiter and thyrotoxic cardiac symptoms in adults. • The most common cause of the syndrome are mutations of the β (beta) form (THRB gene) of the thyroid hormone receptor, of which over 100 different mutations have been documented. • Thyroid hormone resistance syndrome is rare, incidence is variously quoted as 1 in 50,000 or 1 in 40,000 live births.
  • 163. Resistance to thyroid hormone (RTH) • The characteristic blood test results for this disorder can also be found in other disorders (for example TSH-oma (pituitary adenoma), or other pituitary disorders). • The diagnosis may involve identifying mutation of the thyroid receptor, which is present in approximately 85% of cases.
  • 164. Which of the following is a cause of congenital nephrogenic diabetes insipidus? a) Hypercalcemia. b) Hypokalemia . c) Post obstructive uropathy. d) Vasopressine receptor gene mutation .
  • 165. Vasopressin receptor gene mutation is the cause of primary nephrogenic DI not the acquired form.
  • 166. Congenital nephrogenic diabetes insipidus • Two genetic have been identified that cause nephrogenic diabetes insipidus present at birth. – The first is known as the AVRP2 gene mutation and is responsible for 90% of all cases of congenital diabetes insipidus (although it is still very rare, occurring in an estimated 1 in every 250,000 births). – The remaining 10% of cases of congenital nephrogenic diabetes insipidus are caused by the AQP2 gene mutation, which can affect both males and females
  • 167. Acquired nephrogenic diabetes insipidus • The most common cause is the medication lithium. • Lithium is widely used to treat bipolar disorder • over half of all people on long-term lithium therapy will develop nephrogenic diabetes insipidus. • Stopping treatment with lithium will restore normal kidney function, though in many cases the damage to the kidneys is permanent.
  • 168. Acquired nephrogenic diabetes insipidus • Other causes include: – hypercalcemia – hypokalemia – pyelonephritis – ureteral obstruction
  • 169. Which one of the following disorders of growth is characterized by normal body proportion ? a) Achondroplasia. b) Morquio’s. c) Congenital hypothyroidism. d) Hypopituitarism.